Gray's Anatomy Review - Head and Neck

अब Quizwiz के साथ अपने होमवर्क और परीक्षाओं को एस करें!

Cleft lip, with or without cleft palate, occurs about once in 1000 births. Which of the following is considered to be the most important causative factor in the production of this anomaly? A. Riboflavin deficiency B. Infectious disease C. Mutant genes D. Cortisone administration during pregnancy E. Irradiation

11 C. Whereas all forms of clefts are considered to have a multifactorial etiology, cleft lip in particular seems to have a strong genetic factor. This has been determined using studies of twins. The other listed factors may or may not play a role in the development of a cleft lip, but genetics remains the most important determining factor. GAS 1077; N 46; McM 80

A 34-year-old man who swims regularly presents to your office with an external ear canal infection (otitis externa). The patient coughs during inspection of the external auditory meatus with a speculum. The cough results from the irritation of which nerve that innervates an area of the external auditory meatus? A. Vestibulocochlear B. Vagus C. Trigeminal D. Facial E. Accessory

37 B. The vagus nerve innervates a part of the external auditory meatus and, when stimulated, can trigger a cough reflex in about 20% of people. This is thought to be due to "referred sensation" from the vestibule of the larynx, which is innervated by the vagus nerve. The vestibulocochlear nerve is associated only with the inner ear. The trigeminal nerve does provide some innervation to the external auditory meatus but does not affect the cough reflex as does the vagus nerve. The auricular branch of the facial nerve only provides a small amount of general sensory supply to the external ear; it is not associated with the cough reflex. GAS 956; N 95; McM 60, 61

A 55-year-old woman is diagnosed with a tumor at the base of the skull, resulting in a decrease in tear production. Which of the following nerves is most likely injured? A. Chorda tympani B. Deep petrosal C. Greater petrosal D. Lesser petrosal E. Nasociliary

40 C. The greater petrosal nerve, a parasympathetic branch of the facial nerve, provides innervation to the lacrimal gland in the orbit. The chorda tympani provides innervation to the submandibular and sublingual glands and also taste to the anterior two thirds of the tongue. The deep petrosal nerve carries sympathetic innervation to the blood vessels and mucous glands of the head and neck. The lesser petrosal nerve provides parasympathetic innervation to the parotid gland. The nasociliary nerve provides sensory innervation to the ethmoidal sinuses and the cornea as well as innervation to the skin of the eyelids and superior nose regions. GAS 932-933; N 124; McM 55-57

An 11-year-old boy is examined by an otorhinolaryngologist for his swollen palatine tonsils. The palatine tonsils are located between the anterior and posterior tonsillar pillars. Which of the following muscles form these pillars? A. Levator veli palatini and tensor veli palatini B. Palatoglossus and palatopharyngeus C. Styloglossus and stylopharyngeus D. Palatopharyngeus and salpingopharyngeus E. Superior and middle pharyngeal constrictors

55 B. The palatine tonsils lie in tonsillar beds with muscular (covered with mucosa) anterior and posterior pillars forming the boundaries of the bed. These pillars are formed by the palatoglossal arch, anteriorly, and the palatopharyngeal arch, posteriorly. The anterior pillar, part of the palatoglossal arch, contains the palatoglossus muscle; the posterior pillar, provided by the palatopharyngeal arch, is formed by the palatopharyngeus muscle. GAS 1044; N 57; McM 49

A 34-year-old woman is admitted to the hospital with a large mass at her thyroid gland. Ultrasound examination suggests a benign tumor, which is confirmed with a biopsy. Twenty-four hours following a partial thyroidectomy, in which the inferior thyroid artery was also ligated, the patient speaks with a hoarse voice and has difficulty in breathing on exertion. Which of the following nerves was most likely injured during the surgical procedure? A. Internal branch of superior laryngeal B. Ansa cervicalis C. Ansa subclavia D. Recurrent laryngeal E. External branch of superior laryngeal

63 D. The recurrent laryngeal nerve is the most likely nerve damaged during the surgery because it runs in close proximity to the inferior thyroid artery and is easily injured or transected with the artery if extreme care is not exercised during operative procedures. The recurrent laryngeal nerve innervates the majority of the vocal muscles that open and close the rima glottidis, in addition to providing sensory supply to the larynx below the vocal folds. Even relatively mild trauma to the nerve can result in hoarseness. The internal branch of the superior laryngeal nerve is not in close proximity to the inferior thyroid artery and pierces the thyrohyoid membrane to enter the pharynx. The ansa cervicalis lies lateral to the site of surgery and does not innervate any structures that, if paralyzed, would cause hoarseness. GAS 1020, 1021, 1034, 1069; N 26, 33, 69, 71, 76-78, 82, 127, 131, 206-209, 223, 228-229, 236, 238-239; McM 35-36, 47, 49-50, 78

A 38-year-old man is admitted to the hospital with a large mass in his lower anterior neck. Ultrasound examination reveals a benign tumor of his thyroid gland. Twenty-four hours following a partial thyroidectomy, it was noted that the patient could not abduct the true vocal cords due to a nerve injury during the operation. Which of the following muscles was most likely denervated? A. Posterior cricoarytenoid B. Lateral cricoarytenoid C. Thyroarytenoid D. Arytenoid E. Cricothyroid

67 A. The posterior cricoarytenoids are the only muscles of the larynx that abducts the vocal cords. The remaining answer choices are muscles that act in adduction of the vocal cords. GAS 1061, 1062; N 67, 69, 80, 81, 82, 151; McM 47-49

An 8-year-old boy is admitted to the hospital with a drooping right eyelid (ptosis). The initial diagnosis is Horner's syndrome. Which of the following additional signs on the right side would confirm the diagnosis? A. Constricted pupil B. Dry eye C. Exophthalmos D. Pale, blanched face E. Sweaty face

79 A. Horner's syndrome involves interruption of sympathetic supply to the face. This results in ptosis (drooping eyelid), miosis (constricted pupil), and anhydrosis (lack of sweating) of the face. The eye is lubricated by the lacrimal gland, which secretes in response to parasympathetic stimulation, and would be unaffected. Exophthalmos (protrusion of the globe) is frequently caused by hyperthyroidism and is not present in Horner's syndrome. Loss of sympathetic innervation leads to unopposed vasodilatation of the vessels to the face, leading to flushing rather than paleness. GAS 928-931; N 83, 132; McM 81

A 2-month-old male infant presents with a small pit at the anterior border of his left sternocleidomastoid muscle, with mucus dripping intermittently from the opening. The pit extended to the tonsillar fossa as a branchial fistula. Which of the following embryologic structure(s) is (are) involved in this anomaly? A. Second pharyngeal arch B. Second pharyngeal pouch and groove C. Third pharyngeal pouch D. Thyroglossal duct E. Second pharyngeal pouch and cervical sinus

1 E. The child in this case suffers from a fistula that indicates an open malformation. This implies that the defect must be due to failure of closure for both an internal and an external structure. This excludes the second pharyngeal arch and third pharyngeal pouch from being the answers alone. A branchial fistula results from failure of closure of both the second pharyngeal pouch and the cervical sinus, the cervical sinus being the consolidation of the second through fourth pharyngeal clefts, all being external structures. The thyroglossal duct extends from the thyroid to the tongue and failure of its closure would not result in an external defect. The second pharyngeal groove merges with the third and fourth pharyngeal grooves to form the cervical sinus. Failure of closure of the second groove alone would not present with an open fistula. GAS 1016; N 64; McM 28-35

A 3-month-old male infant is under observation in the pediatric clinic. The patient has congenital hypoparathyroidism, thyroid hypoplasia, and no thymus. Abnormal development of which of the following pharyngeal pouches or arches will most likely produce these defects? A. First and second B. Second and third C. Third and fourth D. Fourth E. Fourth and sixth

10 C. The defect is likely in the development of third and fourth pharyngeal pouches because the superior parathyroid glands are derived from the fourth pouch, whereas the inferior parathyroid glands are derived from the third pouch. In addition, the third pouch gives rise to the thymus, and the parafollicular cells of the thyroid gland are derived from the fourth pharyngeal pouch. The first pouch gives rise to the tympanic membrane and cavity. The second pouch gives rise to the palatine tonsils and tonsillar sinus. GAS 1022; N 78; McM 30

An 8-year-old boy had a mastoidectomy due to an infection that did not respond to antibiotics. Postoperatively he had Bell's palsy (facial paralysis), and one of the features was the accumulation of saliva in the vestibule of his oral cavity and dribble from the corner of his mouth. Which of the following muscles was most likely paralyzed? A. Zygomaticus major B. Orbicularis oculi C. Buccinator D. Levator palpebrae superioris E. Orbicularis oris

100 C. Normally the tonus of the buccinator muscle prevents the accumulation of saliva and foodstuffs in the oral vestibule. Although a lesion of the facial nerve would paralyze the other muscles listed, the buccinator is the predominant muscle of the cheek. GAS 904, 909, 911, 1092; N 24, 25, 46, 48, 50, 57, 65; McM 39-40, 43

A 5-week-old male infant is born without a thymus or inferior parathyroid glands. Which of the following pharyngeal arches is most likely involved? A. First B. Second C. Third D. Fourth E. Fifth

12 C. Absence of the thymus and inferior parathyroid glands would be due to defective development of the third pharyngeal pouch, their normal site of origin. The first pouch gives rise to the tympanic membrane and cavity. The second pouch gives rise to the palatine tonsils and tonsillar sinus. The fourth pharyngeal pouch gives rise to the superior parathyroid glands and the parafollicular cells of the thyroid gland. The fifth pharyngeal pouch contributes to the formation of the parafollicular cells of the thyroid gland. GAS 1022; N 78; McM 30

A 22-year-old woman visits the outpatient clinic with a painless swelling on the right side of her neck. A CT scan examination reveals a well-defined cystic mass at the angle of the mandible, just anterior to the sternocleidomastoid muscle. What is the most likely diagnosis? A. Dermoid cyst B. Inflamed lymph node C. Accessory thyroid tissue D. Thyroglossal duct cyst E. Lateral cervical cyst

14 E. A lateral cervical cyst is caused by remnants of the cervical sinus and would present anterior to the sternocleidomastoid. A dermoid cyst is a cystic teratoma that often occurs near the lateral aspect of the eyebrow. A swollen lymph node is likely to present with pain. Accessory thyroid tissue is normally situated along the route of descent of the thyroglossal duct, either in the posterior tongue or along the midline of the neck. A cyst of the thyroglossal duct would be found in locations similar to where accessory thyroid tissue is found. GAS 1016; N 64; McM 28-35

A 5-day-old infant is diagnosed with a noncommunicating hydrocephalus. Which of the following is most likely to lead to such a condition? A. Obstruction in the circulation of the cerebrospinal fluid (CSF) B. Excess production of CSF C. Increased size of the head D. Disturbances in the resorption of CSF E. Failure of the neural tube to close

15 A. Noncommunicating hydrocephalus, also known as obstructive hydrocephalus, is due to an obstruction to flow of CSF within the ventricular system. Excess production of CSF or disturbed resorption of CSF gives rise to communicating or nonobstructive hydrocephalus. An increased size of the head can occur as a result of hydrocephalus but would not be a causative factor for hydrocephalus. Failure of the neural tube to close may lead to anencephaly or spina bifida, depending on the portion of the tube affected, but would not result in hydrocephalus. GAS 877-878; N 147; McM 72, 81

A 5-day-old infant was born with a normal Apgar score. One month later the external acoustic meatus was atretic. Which of the following conditions was the most likely cause of this defect? A. Otic pit did not form B. Development of the first pharyngeal pouch was affected C. Meatal plug did not canalize D. Auricular hillocks did not develop E. The tubotympanic recess degenerated

16 C. A normal APGAR score indicates that the child appeared normal and healthy at birth, based on skin color, heart rate, reflexes, muscle tone, and breathing (Appearance, Pulse, Grimace, Activity, and Respiration). An atretic external acoustic canal occurs due to failure of the meatal plug to canalize, an event that normally occurs in late fetal life. Failure of the otic pit to form results in an absent otic vesicle and absence of the membranous labyrinth. The first pharyngeal pouch gives rise to the tympanic membrane and cavity, and abnormal development would not affect the external acoustic meatus. Failure of the auricular hillocks to develop results in failure of the external ear to develop. A degenerated tubotympanic recess would not lead to an atretic external acoustic meatus. GAS 956; N 95; McM 60

A 50-year-old woman complained of pain over her chin and lower lip. A few days later, small vesicles appeared over the same area and soon began erupting. She was diagnosed with a dermatomal herpes zoster inflammation (shingles). Which of the following nerves was most likely to contain the virus in this case? A. Auriculotemporal B. Buccal C. Lesser petrosal D. Mental E. Infraorbital

17 D. The chin and lower lip area are supplied by the mental nerve, a branch of the inferior alveolar nerve, which in turn is a branch of the mandibular division of the trigeminal nerve (CN V3). The auriculotemporal nerve supplies the TMJ, the temporal region, the parotid gland, and the ear. The buccal nerve is sensory to the internal surface of the cheek. The lesser petrosal nerve is a parasympathetic nerve and would not be affected by herpes zoster, a disease of the dorsal root ganglia. The infraorbital nerve provides sensory innervation to the upper lip. GAS 986; N 50; McM 38

A 68-year-old woman is suffering from excruciating, sudden bouts of pain over the area of her midface. Physical examination indicates that she has tic douloureux (trigeminal neuralgia). Which ganglion is the location of the neural cell bodies of the nerve mediating the pain? A. Geniculate B. Trigeminal (semilunar or Gasserian) C. Inferior glossopharyngeal D. Otic E. Pterygopalatine

18 B. The trigeminal ganglion, also known as the semilunar or Gasserian ganglion, is the location of the sensory neuron cell bodies of the trigeminal nerve (CN V). Tic douloureux (trigeminal neuralgia) is a condition in which pain occurs over the area of distribution of trigeminal nerve branches. The geniculate ganglion is found on the facial nerve (CN VII) and receives sensory fibers for taste and transmits preganglionic parasympathetic fibers. The inferior glossopharyngeal ganglion is part of the glossopharyngeal nerve (CN IX), not the trigeminal nerve, and is not the site of the cell bodies mediating the pain. The otic ganglion, located on the mandibular division of the trigeminal nerve, contains postganglionic parasympathetic cell bodies for parotid secretion. The pterygopalatine ganglion, located in the pterygopalatine fossa, also contains postganglionic parasympathetic cell bodies for lacrimation and mucosal secretion. GAS 921; N 123; McM 75

A 17-year-old girl is admitted to the hospital with signs of cavernous sinus thrombosis, as revealed by radiographic and physical examinations. Thrombophlebitis in the "danger area" of the face can spread to the cavernous sinus and involve the ophthalmic branch of the trigeminal nerve. Which of the following symptoms will most likely be present during physical examination? A. Pain in the hard palate B. Anesthesia of the upper lip C. Pain from the eyeball D. Pain over the lower eyelid E. Tingling sensation over the buccal region of the face

19 C. The ophthalmic branch of the trigeminal nerve (CN V1) supplies sensory innervation to the eyeball, leading to pain when damaged. Pain in the hard palate and lower eyelid and anesthesia of the upper lip would be carried by the maxillary branch of the trigeminal nerve (CN V2). Paraesthesia over the buccal portion of the face would be mediated by the maxillary division of the trigeminal nerve. GAS 914; N 52; McM 75

A 2-day-old female infant was born with a cleft palate. The major portion of the palate develops from which of the following embryonic structures? A. Lateral palatine process B. Median palatine process C. Intermaxillary segment D. Median nasal prominences E. Frontonasal eminence

2 A. The largest part of the palate is formed by the secondary palate, which is embryologically derived from the lateral palatine processes. The median palatine process gives rise to the smaller primary palate, located anteriorly. The intermaxillary segment gives rise to the middle upper lip, premaxillary part of the maxilla, and the primary palate. The median nasal prominences merge with each other and the maxillary prominences to give rise to the intermaxillary segment. The frontonasal eminence gives rise to parts of the forehead, nose, and eyes. GAS 1089, 1077; N 62, 64; McM 1, 9

A 34-year-old man is admitted to the hospital with severe headaches, dizziness, and vomiting. Imaging studies reveal a tumor at the hypoglossal canal. Which of the following muscles will most likely be affected? A. Geniohyoid B. Mylohyoid C. Palatoglossus D. Genioglossus E. Thyrohyoid

20 D. A tumor at the hypoglossal canal would compress the hypoglossal nerve (CN XII) and affect the genioglossus, a muscle it supplies. The palatoglossus is innervated by the vagus nerve (CN X), and the thyrohyoid is innervated by the ansa cervicalis (C1 to C3). The geniohyoid is supplied by C1, which runs with the hypoglossal nerve after it passes through the hypoglossal canal, and would therefore be unaffected. The mylohyoid is supplied by the nerve to mylohyoid, a branch of the mandibular division of the trigeminal nerve (CN V3). GAS 1097-1098; N 59; McM 48

A 45-year-old woman is admitted to the hospital with severe headaches, dizziness, and vomiting. Radiologic examination reveals an intracranial tumor. Upon physical examination the patient has dryness of the nasal and paranasal sinuses, loss of lacrimation, and loss of taste from the anterior two thirds of the tongue. Which of the following structures is most likely involved with the tumor? A. Auriculotemporal nerve B. Lesser petrosal nerve C. Facial nerve D. Inferior salivatory nucleus E. Pterygopalatine ganglion

21 C. The superior salivatory nucleus is the autonomic nucleus for the facial nerve (CN VII). Parasympathetic fibers carried by the greater petrosal branch of the facial nerve are responsible for supply of the lacrimal gland and sinuses, via the pterygopalatine ganglion. The geniculate ganglion contains the cell bodies for taste from the anterior two thirds of the tongue carried by the chorda tympani branch of the facial nerve. This branch also carries the parasympathetic supply for the submandibular and sublingual salivary glands. The auriculotemporal nerve provides sensory innervation to the temporal regions of the head, the TMJ, and general sensation from the ear. The inferior salivatory nucleus provides preganglionic parasympathetic fibers carried by the glossopharyngeal nerve (CN IX) that synapse in the otic ganglion, providing parotid stimulation. The pterygopalatine ganglion includes fibers that innervate only lacrimation and the nasal sinuses, but not taste on the anterior two thirds of the tongue (GAS Fig. 8-84). GAS 911; N 124; McM 39

A 17-year-old girl has suffered from a painful discoloration of her hand for the past year. A magnetic resonance imaging (MRI) scan revealed "thoracic outlet" syndrome. Which symptom would most likely result from this syndrome? A. Problems with respiration because of pressure on the phrenic nerve B. Reduced blood flow to the thoracic wall C. Reduced venous return from the head and neck D. Numbness in the upper limb E. Distention of the internal jugular vein

22 D. Thoracic outlet syndrome is characterized by the presence of a cervical rib, accessory muscles, or connective tissue bands that constrict the limited dimensions of the thoracic outlet. The cervical rib is usually located on the C7 vertebra and can impinge on the brachial plexus, resulting in loss of some feeling to the upper limb. There would be no impingement on the phrenic nerve because it leaves C3 to C5 directly parallel with the vertebral column. The syndrome does not include reduction of blood flow to the thoracic wall because of extensive anastomoses between the vessels that supply blood to the anterior thoracic wall. Venous return from the head and neck is mainly through the internal jugular vein and would not be affected because of this vein's location near the midline of the body; thus, it would not be occluded or distended. GAS 150; N 182, 186; McM 216

A 31-year-old woman is admitted to the hospital after an automobile collision. An MRI examination reveals a large hematoma inferior to the right jugular foramen. Physical examination reveals right pupillary constriction (miosis), ptosis of the eyelid, and anhydrosis (loss of sweating) of the face. Which of the following ganglia is most likely affected by the hematoma? A. Submandibular B. Trigeminal (semilunar or Gasserian) C. Superior cervical D. Geniculate E. Ciliary

23 C. The superior cervical ganglion (SCG), which is the uppermost part of the sympathetic chain, supplies sympathetic innervation to the head and neck. The usual symptoms for SCG injury, commonly known as Horner's syndrome, are miosis, ptosis, and anhydrosis in the head and neck region. Postganglionic sympathetic nerves usually run alongside the arteries leading into the head and neck region. The submandibular ganglion does not carry sympathetic nerves to areas of the head and neck. The trigeminal ganglion includes only cell bodies from afferent sensory nerves from the head. The geniculate ganglion includes cell bodies for taste sensation from the anterior two thirds of the tongue, carried by the facial nerve; it also transmits parasympathetic innervation to many sections of the head and face. The ciliary ganglion provides parasympathetic innervation to the eye and also has some sympathetic fibers coursing through but not synapsing; thus, it would not account for the symptoms of the face. GAS 1036; N 132; McM 47

A 35-year-old man is admitted to the hospital with severe headaches. A CT scan evaluation reveals a tumor in the infratemporal fossa. Physical examination reveals loss of general sensation from the anterior two thirds of his tongue, but taste and salivation are intact. Which of the following nerves is most likely affected by the tumor? A. Lingual proximal to its junction with the chorda tympani B. Chorda tympani C. Inferior alveolar D. Lesser petrosal E. Glossopharyngeal

24 A. The chorda tympani joins the lingual nerve in the infratemporal fossa, and a lesion to the lingual nerve before it joins the chorda tympani would account for the loss of general sensation, with no loss to the special sense of taste and saliva production. If the chorda tympani were injured, the patient would present with a loss of taste (from the anterior two thirds of tongue) and a decrease in saliva production because the submandibular and sublingual salivary glands would be denervated. The inferior alveolar nerve provides sensory innervation to the mandibular teeth, but no such loss is present. The lesser petrosal nerve innervates postganglionic neurons supplying the parotid gland, but no loss of salivation is present. The glossopharyngeal nerve provides taste innervation to the posterior third of the tongue and sensation related to the gag reflex, but there is no deficit present in this patient. GAS 1101; N 46; McM 76

A 70-year-old man is admitted to the hospital with chronic headache and enlarged lymph nodes. A CT scan shows a tumor at the jugular foramen. Which of the following would be the most likely neurologic deficit? A. Loss of tongue movements B. Loss of facial expression C. Loss of sensation from the face and the scalp D. Loss of hearing E. Loss of gag reflex

25 E. The jugular foramen is the route of exit for three nerves (glossopharyngeal [CN IX], vagus [CN X], and spinal accessory [CN XI] nerves) and one vein (internal jugular vein) from the cranial cavity. The glossopharyngeal nerve provides the sensory input for the gag reflex, whereas the vagus nerve provides the motor output. Nerve compression within this foramen would lead to a loss of both systems and thus no gag reflex. Tongue movements are almost entirely supplied by the hypoglossal nerve (CN XII), which exits the skull through the hypoglossal canal. The facial nerve (CN VII) innervates the muscles of the face and would not be affected by this injury. Loss of sensation from the face and scalp would be present only if there was involvement of the trigeminal nerve. Loss of hearing would be present with any compression of the vestibulocochlear nerve (CN VIII). The vestibulocochlear nerve was formally called the auditory nerve but vestibulocochlear clearly refers to its dual function and hearing. GAS 870; N 13; McM 11

A 40-year-old unconscious man is admitted to the emergency department after being hit in the head with a baseball. A CT scan examination reveals a fractured pterion and an epidural hematoma. Branches of which of the following arteries are most likely to be injured? A. External carotid B. Superficial temporal C. Maxillary D. Deep temporal E. Middle meningeal

26 E. The middle meningeal artery is a branch of the maxillary artery and courses between the dura mater and skull close to the area of the pterion. Any fracture or impact trauma to this location typically results in a laceration of the middle meningeal artery resulting in an epidural hematoma. The external carotid artery ends behind the mandible by dividing into the maxillary and the superficial temporal arteries, and neither of these arteries directly affects the meninges of the brain. The deep temporal arteries do not penetrate the bony skull and thus would not contribute to an epidural hematoma (GAS Fig. 8-33). GAS 990; N 102; McM 42

An unconscious 48-year-old woman is admitted to the hospital. CT scan reveals a tumor in her brain. When she regains consciousness, her right eye is directed laterally and downward, with complete ptosis of her upper eyelid, and her pupil is dilated. Which of the following structures was most likely affected by the tumor to result in these symptoms? A. Oculomotor nerve B. Optic nerve C. Facial nerve D. Ciliary ganglion E. Superior cervical ganglion

27 A. An injury to the oculomotor nerve (CN III) would cause the eye to point downward and laterally due to the unopposed contractions of the muscles innervated by the trochlear (CN IV) and abducens (CN VI) nerves (superior oblique and lateral rectus muscles, respectively). The oculomotor nerve also provides innervation to the levator palpebrae superioris; thus, any injury will cause complete ptosis or inability to raise the eyelid. The constriction of the pupil is provided by parasympathetic nerves via the oculomotor nerve. The optic nerve is responsible only for the sensory aspect of light via the retina in the eye. The facial nerve innervates the facial muscles, including the orbicularis oculi, which closes the eylid for the blink reflex. The ciliary ganglion could be damaged in this patient, but the loss of parasympathetic supply will not adequately explain the ptosis of the eyelid. The superior cervical ganglion provides sympathetic innervation to the head and neck including the smooth muscle in the upper lid (superior tarsal muscle of Müller), but no loss of sympathetics is evident in this patient. GAS 901-902; N 55; McM 55, 57, 81

A 55-year-old man is admitted to the hospital after an injury sustained at work in a factory. He presents with severe scalp lacerations, which were sutured. After 3 days the wound is inflamed, swollen, and painful. Between which tissue layers is the infection most likely located? A. The periosteum and bone B. The aponeurosis and the periosteum C. The dense connective tissue and the aponeurosis D. The dense connective tissue and the skin E. The dermis and the epidermis

28 B. The scalp is divided into five layers: skin, dense connective tissue, aponeurosis, loose connective tissue, and periosteum. A handy mnemonic is SCALP (skin, connective tissue, aponeurosis, loose connective tissue, and periosteum). Typically, infections will be located in the loose connective tissue because of the ease with which infectious agents spread via the many veins located in this region. This area is usually referred to as the "danger zone" of the scalp mainly because scalp infections here can be transmitted into the skull via emissary veins, then via diploic veins of the bone to the cranial cavity. The periosteum and bone are tightly bound together; thus, it is not likely to find infections between these layers. The areas between the dense connective tissue and aponeurosis and between the connective tissue and the skin layers do not include connecting veins but mainly superficial veins of the head. The skin provides a very strong barrier against infections; the epidermis and dermis layers are rarely seen separated, and thus the likelihood of an infection between these areas would be rare. GAS 922; N 3, 25; McM 5, 81, 82

A 36-year-old woman is admitted to the hospital with severe head injuries after a car crash. During neurologic examination her uvula is found to be deviated to the right. Which nerve is most likely affected to result in this deviation? A. Left vagus B. Right vagus C. Right hypoglossal D. Left glossopharyngeal E. Right glossopharyngeal

29 A. An injury to the left vagus nerve (CN X) would cause the uvula to become deviated to the right. This is because the vagus nerve innervates the musculus uvulae muscle that makes up the core of the uvula. If only one side is effectively innervated, contraction of the active muscle will deviate the uvula to the contralateral side of the injury (ipsilateral side of the uninjured vagus nerve). In addition, the intact levator veli palatini will pull the uvula to the intact side. The right and left hypoglossal nerves innervate the tongue muscles and would not affect the uvula. The glossopharyngeal nerve supplies sensory innervation to the oropharynx and nasopharynx, but not motor innervation to these areas. GAS 901; N 71, 127; McM 47

A 3-day-old male infant has a small area of the right iris missing, and a diagnosis of coloboma of the iris is made. Which of the following is the most likely embryologic cause of the coloboma? A. Failure of the retinal/choroid fissure to close B. Abnormal neural crest formation C. Abnormal interactions between the optic vesicle and ectoderm D. Posterior chamber cavitation E. Weak adhesion between the inner and outer layers of the optic vesicle

3 A. A coloboma of the iris is caused by failure of the retinal fissure to close during the sixth week. Abnormal neural crest formation would lead to abnormal development of choroid, sclera, and cornea because these are derived from neural crest cells. Abnormal interaction between the optic vesicle and ectoderm would lead to abnormal development of the entire eye because a lens placode may fail to develop or develop abnormally. The iris would not be affected by abnormal development of the posterior chamber. Weak adhesion between the layers of the optic vesicle leads to congenital retinal detachment. GAS 947; N 83; McM 54

A 22-year-old man is admitted to the emergency department and intubated. An endotracheal tube is passed through an opening between the vocal folds. What is the name of this opening? A. Piriform recess B. Vestibule C. Ventricle D. Vallecula E. Rima glottidis

30 E. The rima glottidis is the opening between the vocal folds and the arytenoid cartilages on the right and left sides. The piriform recess is the recess lateral to the laryngeal opening of the laryngopharynx. The vestibule is the region between the epiglottis and rima glottidis. The ventricle is the area between the true and false vocal cords. The vallecula is a bilateral recess anterior to the epiglottis just posterior to the tongue in the laryngopharynx (GAS Fig. 8-217). GAS 1060; N 80; McM 49

A 55-year-old man has a complaint of left-sided tooth pain in his upper jaw. A dental examination reveals no abnormalities of his teeth. During physical examination tapping on his left maxilla elicits sharp pain on the left side of his face, including his maxillary teeth on that side. The patient reports that he has no allergies. Which of the following conditions will be the most likely diagnosis? A. Sphenoid sinusitis B. Anterior ethmoidal sinusitis C. Posterior ethmoidal sinusitis D. Maxillary sinusitis E. Frontal sinusitis

31 D. Maxillary sinusitis is an infection of the maxillary sinus, which is located in the body of the maxillary bone. Sharp pain can be a major symptom of maxillary sinusitis. The difference between the remaining answer choices is the location of the sinus. The sphenoid sinus is located in the posterior nasopharynx. The ethmoidal sinuses are located laterosuperiorly to the nasal septum. The frontal sinus is located in the frontal bone in the anterior part of the face. GAS 1074-1076; N 42-44; McM 43, 76

A 70-year-old man is admitted to the hospital with severe headaches. During the physical examination he has difficulty coughing and swallowing. An MRI scan shows a tumor affecting a cranial nerve. Which nerve is most likely affected? A. Mandibular B. Maxillary C. Glossopharyngeal D. Vagus E. Hypoglossal

32 D. The vagus nerve is responsible for sensation in the mucosa of the larynx down to the level of the vocal folds, and also motor innervation of the muscles that initiate a cough reflex and swallowing (motor). The mandibular division of the trigeminal nerve provides sensory innervation to the mouth and lower and lateral face and motor innervation to the muscles of mastication. The maxillary division of the trigeminal nerve provides only sensory innervation to the midfacial region surrounding the maxillary bone. The glossopharyngeal nerve provides sensory innervation to the pharynx (gag reflex) and motor innervation to the stylopharyngeus muscle. The hypoglossal nerve innervates most of the muscles of the tongue and is not associated with the cough reflex. GAS 900, 1052-1053; N 71; McM 45

A 7-year-old boy with a high fever is brought to the pediatrician. During physical examination the patient complains of pain in his ear. His throat appears red and inflamed, confirming the diagnosis of pharyngitis. Which of the following structures provided a pathway for the infection to spread to the tympanic cavity (middle ear)? A. Choanae B. Internal acoustic meatus C. External acoustic meatus D. Pharyngotympanic tube E. Pharyngeal recess

33 D. The pharyngotympanic (eustachian) tube connects the middle ear and the nasopharynx and is the conduit for spreading infections. The choanae are the openings between the nasal cavity and the nasopharynx, but they are not involved in spreading infection. The internal and external auditory meatuses are not directly associated with the middle ear but are associated with the inner and outer ear, respectively. The pharyngeal recess is a slitlike opening located behind the entrance to the auditory tube in the nasopharynx. Adenoids, enlarged masses of lymphoid tissue, can develop there. GAS 956, 958, 960; N 94, 100; McM 60, 61

A 33-year-old woman is unconscious when she is admitted to the hospital after she fell, hitting her head. The physician in the emergency department performs a pupillary light reflex test. The integrity of which of the following nerves is being checked? A. Optic and facial B. Optic and oculomotor C. Maxillary and facial D. Ophthalmic and oculomotor E. Ophthalmic and facial

34 B. The optic and oculomotor nerves are responsible for the sensory and motor portions, respectively, of the pupillary light reflex. The optic nerve would include the sensory portion, but the facial nerve only closes the eyelid and does not affect the pupil. The maxillary division of the trigeminal nerve only provides sensory innervation to the skin surrounding the maxillary bone. The ophthalmic division of the trigeminal nerve provides sensory innervation to the cornea for the corneal reflex, but not the light reflex. GAS 901; N 88; McM 54

A 48-year-old man complains of diplopia (double vision). On neurologic examination he is unable to adduct his left eye and lacks a corneal reflex on the left side. Where is the most likely location of the lesion resulting in the symptoms? A. Inferior orbital fissure B. Optic canal C. Superior orbital fissure D. Foramen rotundum E. Foramen ovale

35 C. The superior orbital fissure is the opening that allows the passage of the oculomotor nerve and the trochlear nerve; the lacrimal, frontal, and nasociliary branches of ophthalmic division of the trigeminal nerve; the abducens nerve; the superior and inferior divisions of the ophthalmic vein; and the sympathetic fibers from the cavernous plexus. The sensory and motor components of the corneal reflex are the ophthalmic division of the trigeminal nerve and the oculomotor nerve, whereas the eye impairment is due to a lesion to the oculomotor nerve, all of which are transmitted through the superior orbital fissure. The inferior orbital fissure contains the maxillary division of the trigeminal nerve, infraorbital vessels, and branches of the pterygopalatine ganglion. The optic canal contains the ophthalmic artery and optic nerve, in addition to sympathetic fibers. The foramen rotundum contains the maxillary nerve. The foramen ovale contains the lesser petrosal nerve, the mandibular division of the trigeminal nerve, the accessory middle meningeal artery, and the emissary veins. GAS 901, 934; N 85; McM 23, 25

A 34-year-old man complains of hyperacusis (sensitivity to loud sounds). Injury to which of the following cranial nerves is most likely responsible? A. Hypoglossal B. Facial C. Accessory D. Vagus E. Glossopharyngeal

36 B. The facial nerve innervates the stapedius muscle, which is responsible for limiting movement of the stapes, thereby reducing the intensity of the sound entering the inner ear. The hypoglossal nerve innervates tongue muscles; the spinal accessory nerve supplies the trapezius and sternocleidomastoid muscles; the vagus nerve does not provide any innervation for sound in the ear; and the glossopharyngeal nerve only supplies sensation to the posterior third of the tongue, pharynx (gag reflex), middle ear cavity, and tympanic membrane, and muscle innervation to the stylopharyngeus muscle. GAS 895, 898-899; N 96; McM 60, 61

A 29-year-old woman underwent a thyroidectomy. Postoperatively, the patient presented with hoarseness. Which of the following nerves was most likely injured during the operation? A. Internal laryngeal B. External laryngeal C. Recurrent laryngeal D. Superior laryngeal E. Glossopharyngeal

38 C. The recurrent laryngeal nerve supplies most of the motor innervation to the larynx and sensation below the true vocal folds. The thyroid gland and the recurrent laryngeal nerve are in close proximity and thus the nerve is the most likely to be injured with a thyroidectomy. Injury to the recurrent laryngeal nerve can result in speech defects, including hoarseness. The superior laryngeal nerve has two branches: the internal laryngeal nerve innervates the mucous membranes of the larynx above the vocal folds, and the external laryngeal nerve innervates the cricothyroid muscle, which tenses the vocal folds. The glossopharyngeal nerve is located superiorly to the true vocal folds and would not be affected by this procedure. GAS 1021; N 78; McM 49

A 48-year-old man presents with a constricted right pupil that does not react to light. His left pupil and vision in both eyes are normal. These findings are most likely due to a lesion involving which of the following right-sided structures? A. Oculomotor nerve B. Superior cervical ganglion C. Nervus intermedius D. Edinger-Westphal nucleus E. Trigeminal (semilunar, Gasserian) ganglion

39 B. The superior cervical ganglion provides sympathetic innervation to the face and neck regions. Sympathetics travel along the branches of the internal carotid artery, and one result of stimulation of these nerves is to dilate the pupil during a sympathetic response ("flight or fight"). The oculomotor nerve would not affect the dilation of the pupil; rather, its stimulation results in the constriction (parasympathetic nerves). The nervus intermedius is the parasympathetic component to the facial nerve and affects only lacrimation of the eye. The Edinger-Westphal nucleus is the location of the cell bodies of the preganglionic parasympathetic neurons that are carried by the oculomotor nerve (not sympathetics). The trigeminal ganglion only provides sensory innervation to the face and eye but has no motor effect on the pupil. GAS 931, 1036; N 132; McM 44

Early closure of the fontanelles of the infant skull can result in compression of the brain, restricting brain growth. Which of the following fontanelles is located at the junction of the sagittal and coronal sutures and at what age does this fontanelle typically close? A. Posterior fontanelle, which closes at about 2 years B. Mastoid fontanelle, which closes at about 16 months C. Lambdoid fontanelle, which closes at 8 months to 1 year D. Sphenoidal fontanelle, which closes at 3 years E. Anterior fontanelle, which closes at 18 months

4 E. The anterior fontanelle is located at the junction of the sagittal and coronal sutures and closes at around 18 months of age. The posterior fontanelle is located at the junction of the sagittal suture and lambdoid suture, and it closes at around 2 to 3 months. The mastoid fontanelle is located at the junction of the squamous suture and the lambdoid suture, and it closes at the end of the first year. The sphenoidal fontanelle is located at the junction of the squamous suture and the coronal suture and closes at around 2 to 3 months. There is a lambdoid suture but not a lambdoid fontanelle. GAS 860, 864; N 9; McM 14

A 24-year-old man is admitted to the hospital after a street fight. Radiographic examination reveals an inferior (blow-out) fracture of the orbit. Orbital structures would most likely be found inferiorly in which of the following spaces? A. Ethmoidal sinus B. Frontal sinus C. Maxillary sinus D. Nasal cavity E. Sphenoidal sinus

41 C. The maxillary sinus is located directly inferior to the orbit. Any trauma to the inferior bony wall of the orbit will likely displace the orbital structures in the compartment to the space below the orbit (maxillary sinus). In most people the sclera of the eyeball is stronger than in the orbital floor, so a blow-out fracture is more likely than rupture of the eyeball. The ethmoidal sinus is located superiorly and medially to the orbit, whereas the frontal sinus is located superiorly to the orbit. The nasal cavity is toward the midline and is not inferior to the orbit. The sphenoidal sinus is deeper into the facial region but is not inferior to the orbit. GAS 1074, 1076; N 43; McM 18

A 35-year-old woman is hospitalized due to cavernous sinus thrombosis resulting from an infection on her face. Which of the following is the most direct route for spread of infection from the face to the cavernous sinus? A. Pterygoid venous plexus B. Superior ophthalmic vein C. Frontal venous plexus D. Basilar venous plexus E. Parietal emissary vein

42 B. The superior ophthalmic vein drains directly into the cavernous sinus. The danger area of the face is located in the triangular region from the lateral angle of the eye to the middle of the upper lip, near the nose, and is drained by the facial vein. The facial vein communicates directly with the cavernous sinus through the superior ophthalmic vein. The pterygoid venous plexus communicates with the cavernous sinus through the inferior ophthalmic vein, but it is not directly connected to the cavernous sinus. The basilar venous plexus connects the inferior petrosal sinuses and communicates with the internal vertebral venous plexus. The parietal emissary veins and frontal venous plexus do not communicate directly with the cavernous sinus. GAS 888; N 73; McM 51

A 7-year-old boy was suffering from a severe infection of the middle ear (otitis media), which spread to the mastoid air cells (mastoiditis). Surgery was required but resulted in the following: right corner of the mouth drooping, unable to close his right eye, food collection in his right oral vestibule. Which nerve was injured? A. Glossopharyngeal B. Vagus C. Facial D. Maxillary division of the trigeminal nerve E. Mandibular division of the trigeminal nerve

43 C. A lesion of the facial nerve is likely to lead to the symptoms described (drooping mouth, unable to close right eye, and food collection in the oral vestibule) because the muscles of facial expression are paralyzed. There is a bony prominence over the facial nerve located on the medial wall of the middle ear. Because of its close proximity, the facial nerve can be very rarely damaged due to otitis media. The other nerves listed are not located in close proximity to the middle ear and, if injured, would not present with the symptoms described. GAS 960; N 94; McM 60, 61

The arterial circle (of Willis) contributes greatly to cerebral arterial circulation when one primary artery becomes occluded by atherosclerotic disease. Which of the following vessels does not contribute to the circle? A. Anterior communicating artery B. Posterior communicating artery C. Middle cerebral artery D. Internal carotid artery E. Posterior cerebral artery

44 C. The arterial circle (of Willis) receives its blood supply from the internal carotid and vertebral arteries. The actual circle is formed by the bifurcation of the basilar, posterior cerebral, posterior communicating, internal carotid, anterior cerebral, and anterior communicating arteries. The middle cerebral artery is the lateral continuation of the internal carotid artery and therefore not part of the arterial circle. Although it receives its blood supply from the arterial circle (of Willis), it does not actually form any part of the circle (GAS Fig. 8-38A). GAS 883; N 141; McM 68

A 45-year-old woman is admitted to the hospital for severe ear pain. Physical examination reveals chronic infection of the mastoid air cells (mastoiditis). The infection can erode the thin layer of the bone between the mastoid air cells and the posterior cranial fossa and spread most commonly into which of the following venous structures? A. Superior sagittal sinus B. Inferior sagittal sinus C. Straight sinus D. Cavernous sinus E. Sigmoid sinus

45 E. The sigmoid venous sinus empties into the internal jugular vein and drains the cranial vault. It runs along the posterior cranial fossa near the suture between the temporal and occipital bones just lateral to the mastoid air cells. The superior sagittal sinus lies within the superior aspect of the longitudinal fissure, between the two cerebral hemispheres. The inferior sagittal sinus runs inferior to the superior sagittal sinus within the falx cerebri and joins the great cerebral vein (of Galen) to form the straight sinus. The straight sinus drains the great cerebral vein (of Galen) into the confluence of sinuses. The cavernous sinus is located within the middle cranial fossa and receives the ophthalmic veins, the greater petrosal sinus, and other venous vessels (GAS Fig. 8-44). GAS 887; N 99; McM 53

A 63-year-old man with hearing loss in his left ear complains of a loss of taste and drooling from the left side of his mouth. A CT scan shows a tumor compressing the nerve exiting the skull through which of the following openings? A. Foramen ovale B. Foramen rotundum C. Internal acoustic meatus D. Jugular foramen E. Superior orbital fissure

46 C. The tumor is compressing the facial nerve, which runs through the internal acoustic meatus along with the vestibulocochlear nerve. The facial nerve provides the sensation of taste to the anterior two thirds of the tongue via the chorda tympani and also mediates all of the facial muscles, except the muscles of mastication. The mandibular branch of the trigeminal nerve courses through the foramen ovale and mediates motor to the muscles of mastication and sensory to the lower third of the face. The maxillary branch of the trigeminal branch passes through the foramen rotundum and is sensory to the middle third of the face. The jugular foramen has the glossopharyngeal, vagus, and accessory nerves coursing through it. Finally, the superior orbital fissure has the ophthalmic branch of the trigeminal nerve coursing through it, along with the oculomotor, trochlear, and abducens nerves. GAS 898-899; N 94; McM 61

A 70-year-old man has a biopsy of a growth on his lower lip. The biopsy reveals a squamous cell carcinoma. Which lymph nodes will most likely be first involved in the spread of the cancer cells? A. Occipital B. Parotid C. Retropharyngeal D. Jugulodigastric E. Submental

47 E. The submental lymph nodes drain approximately the anterior two thirds of the mouth and tongue, including the lower lips. The occipital nodes serve the inferoposterior aspect of the head. The parotid nodes lie anterior to the ear and serve the region of the lateral aspect of the eye, the parotid gland, and anterior ear. The retropharyngeal nodes lie posterior to the pharynx and drain the posterior aspect of the throat and pharynx. The jugulodigastric node is a large node posterior to the parotid gland and just below the angle of the mandible, and it receives lymph from much of the face and scalp and is commonly enlarged in tonsillitis. GAS 920; N 75; McM 13

A 54-year-old man is admitted to the hospital due to severe headaches. A CT examination reveals an internal carotid artery aneurysm inside the cavernous sinus. Which of the following nerves would be typically affected first? A. Abducens nerve B. Oculomotor nerve C. Ophthalmic nerve D. Maxillary nerve E. Trochlear nerve

48 A. The abducens nerve would be affected first due to aneurysmal dilation of the internal carotid artery (ICA) because the nerve runs in closest proximity to the artery within the cavernous sinus. The other nerves running in the wall of the cavernous sinus are the oculomotor nerve, trochlear nerve, and both the maxillary and ophthalmic branches of the trigeminal nerve. Each of these nerves, however, courses along, or within, the lateral walls of the cavernous sinus and may not be immediately affected by an aneurysm of the ICA (GAS Fig. 8-45). GAS 888; N 105; McM 51

A 24-year-old man had a third molar (wisdom tooth) extracted from his lower jaw. This resulted in the loss of general sense and taste sensation from the anterior two thirds of the tongue. This loss was most likely due to injury of which of the following nerves? A. Auriculotemporal B. Chorda tympani C. Lingual D. Mental E. Inferior alveolar

49 C. The lingual nerve is the most likely nerve damaged because there is loss both of taste and general sensory supply to the anterior two thirds of the tongue, which is innervated by the lingual nerve, which at this point has been joined by the chorda tympani. The chorda tympani would be a likely choice; however, it carries only taste and does not mediate other general sensation to the tongue. The auriculotemporal nerve is a posterior branch of the mandibular division of the trigeminal nerve and innervates skin near the ear and temporal region. The mental nerve is the terminal branch of the inferior alveolar nerve and innervates the skin of the chin. GAS 1101; N 50; McM 35

A 3-year-old boy is admitted to the hospital because of a soft anterior midline cervical mass. When he is asked to protrude his tongue, the mass in the neck is observed to move upward. Which of the following is the most likely diagnosis? A. A thyroglossal duct cyst B. Defect in sixth pharyngeal arch C. A branchial cyst D. Cystic fistula of the third pharyngeal arch E. Defect in first pharyngeal arch

5 A. Thyroglossal duct cysts occur due to retention of a remnant of the thyroglossal duct along the path followed by the descending thyroid gland during development. The path begins at the foramen cecum of the tongue and descends in the midline to the final position of the thyroid. The sixth pharyngeal arch provides origin to muscles and cartilage of the neck and would produce a midline mass connected to the tongue. A branchial cyst or fistula would not be present in the midline. The first pharyngeal arch gives rise to muscles of mastication and the malleus and incus. The third pharyngeal arch provides origin to the stylopharyngeus muscle and hyoid bone. GAS 1020; N 64; McM 33

A 56-year-old woman is admitted to the hospital with rheumatoid arthritis of her temporomandibular joint (TMJ) and severe ear pain. Which of the following nerves is most likely responsible for conducting the pain sensation? A. Facial B. Auriculotemporal C. Lesser petrosal D. Vestibulocochlear E. Chorda tympani

50 B. The auriculotemporal nerve is a posterior branch of the mandibular division of the trigeminal nerve. It encircles the middle meningeal artery and courses medially to the TMJ and then ascends up near the auricle. Because this nerve supplies the TMJ and skin of the external auditory canal, pain from the joint can be referred to the ear as in this case. The facial nerve courses over the ascending ramus of the mandible, passing superficial to the masseter muscle and below the TMJ through the parotid gland, and would not be involved in this problem. The lesser petrosal nerve courses through the middle cranial fossa and exits through the foramen ovale, where it joins the otic ganglion. The vestibulocochlear nerve exits the cranial cavity through the internal acoustic meatus and innervates structures in the inner ear. Finally, the chorda tympani is a branch of the facial nerve and joins the mandibular division of the trigeminal nerve anterior to the TMJ. GAS 985-986; N 50; McM 29

51 Where is the location of the postganglionic parasympathetic neural cell bodies that directly innervate the parotid gland? A. Trigeminal (semilunar, Gasserian) ganglion B. Inferior salivatory nucleus C. Superior cervical ganglion D. Otic ganglion E. Submandibular ganglion

51 D. The otic ganglion is the location of the postganglionic parasympathetic neural cell bodies innervating the parotid gland. The ganglion lies on the mandibular division of the trigeminal nerve near the foramen ovale. The trigeminal ganglion contains cell bodies for neurons innervating sensory aspects of the face. The inferior salivatory nucleus lies within the brainstem and contains preganglionic parasympathetic neurons whose axons pass within the lesser petrosal nerve to the otic ganglion for synapse in the supply of the parotid. The superior cervical ganglion has the cell bodies of postganglionic sympathetic fibers innervating sympathetic structures to the head. The submandibular ganglia contain the cell bodies of postganglionic parasympathetic fibers innervating the sublingual and submandibular salivary glands. GAS 889-900; N 135; McM 33

The arachnoid villi allow cerebrospinal fluid to pass between which two of the following spaces? A. Choroid plexus and subdural space B. Subarachnoid space and superior sagittal sinus C. Subdural space and cavernous sinus D. Superior sagittal sinus and jugular vein E. Epidural and subdural space

52 B. The arachnoid villi are extensions of the arachnoid mater into the superior sagittal sinus. The villi allow for proper drainage of the CSF into the venous bloodstream from the subarachnoid space in which the CSF circulates. These villi are a crucial element in maintaining proper intracranial pressure and circulation of the CSF. GAS 876; N 103; McM 2

A 22-year-old woman is admitted to the hospital with an injury to her eye. The corneal reflex is tested and found to be present. Which of the following nerves is responsible for the afferent limb of this reflex? A. Frontal B. Lacrimal C. Nasociliary D. Oculomotor E. Optic

53 C. The afferent/sensory limb of the corneal (blink) reflex is carried by the nasociliary nerve. It is a branch of the ophthalmic division of the trigeminal nerve. The frontal and lacrimal nerves provide cutaneous supply to parts of the orbit and face, but they do not innervate the cornea. The facial nerve is the efferent limb of the corneal reflex and mediates the closing of both eyes in response to irritation of the cornea. The oculomotor nerve mediates the reopening of the eyes by contraction of the levator palpebrae superioris. The optic nerve also innervates the eye for the sense of vision and is the afferent limb of the pupillary light reflex. GAS 901; N 88; McM 44, 54, 55

A 21-year-old man was brought to the emergency department because of severe epistaxis (nosebleed) from the nasal septum. This area, known as Kiesselbach's (or Little's) area, involves mostly anastomoses between which of the following arteries? A. Ascending palatine and ascending pharyngeal B. Posterior superior alveolar and accessory meningeal C. Lateral branches of posterior ethmoidal and middle meningeal D. Septal branches of the sphenopalatine and superior labial E. Descending palatine and tonsillar branches of the pharyngeal

54 D. Kiesselbach's plexus (also called Little's area) is an anastomosis of four arteries on the anterior nasal septum. The four arteries are the anterior ethmoidal artery, sphenopalatine artery, superior labial artery, and greater palatine artery. The two largest contributors, however, are the septal branches of the sphenopalatine (from the maxillary artery) and superior labial arteries (branches of the facial artery, which in turn is a branch of the external carotid artery) (GAS Fig. 8-239AB). GAS 998, 999; N 40; McM 42

A 35-year-old woman is under general anesthesia. Before laryngeal intubation the rima glottidis is opened by which pair of muscles? A. Posterior cricoarytenoids B. Lateral cricoarytenoids C. Thyroarytenoids D. Transverse arytenoids E. Cricothyroids

56 A. The posterior cricoarytenoid muscles lie on the superoposterior aspect of the lamina of the cricoid cartilage. When these muscles contract, they cause lateral rotation (abduction) of the vocal processes of the arytenoid cartilages, thereby opening the space between the vocal folds, the rima glottidis. The lateral cricoarytenoid is involved with adducting the arytenoid cartilage and closing the rima glottidis. The thyroarytenoid muscles lie alongside either vocal ligament and are also involved in adducting the vocal folds. The transverse arytenoid muscle connects both arytenoid cartilages and also aids in closing the rima glottidis. Finally, the cricothyroid muscle is located on the anterior aspect of the cricoid cartilage and aids in elongation and tensing of the vocal folds, thus raising the pitch of the voice. GAS 1059-1061; N 56; McM 47

A 32-year-old woman asks you what is the soft, thin ridge of tissue that she can feel running forward across the masseter muscle toward her upper lip. You reassure her that is perfectly normal. Which of the following is the most likely structure she is feeling? A. Facial artery B. Maxillary artery C. Parotid duct D. Marginal mandibular branch of the facial nerve E. Facial vein

57 C. The parotid duct, also known as the Stensen's duct, crosses the masseter muscle transversely and extends to the oral cavity where it opens by the upper second molar. The facial artery can be palpated in the groove anterior to the mandibular angle. The facial vein lies anterior to the artery, passing toward the angle of the lips, but does not ascend in close proximity to the masseter. All of the other vessels are located more deeply and cannot be palpated. GAS 911; N 56; McM 40

A 43-year-old man is diagnosed with laryngeal carcinoma. A surgical procedure is performed and the tumor is successfully removed from the larynx. The right ansa cervicalis is anastomosed with the right recurrent laryngeal nerve to reinnervate the muscles of the larynx and restore phonation. Which of the following muscles will most likely be paralyzed after this operation? A. Sternocleidomastoid B. Platysma C. Sternohyoid D. Trapezius E. Cricothyroid

58 C. Because of the surgical division of the ansa cervicalis, the sternohyoid muscle will most likely be paralyzed following this tumor resection. The ansa cervicalis innervates the strap muscles, including the sternohyoid, sternothyroid, and omohyoid muscles. The sternocleidomastoid is innervated by the spinal accessory nerve and will not be involved with this surgery. The platysma is located most superficially on the neck and is innervated by the cervical branch of the facial nerve. The trapezius muscle is also innervated by the spinal accessory nerve and plays no role in ansa cervicalis functions. Finally, the cricothyroid muscle is innervated by the external laryngeal branch of the vagus and would not be affected by the surgery. GAS 1016; N 31; McM 31, 33

A 67-year-old woman is admitted to the emergency department with a severe swelling on the right side of her neck. An MRI examination reveals an abscess. The abscess is surgically removed from the middle of the posterior cervical triangle on the right side. During recovery the patient notices that her shoulder droops and she can no longer raise her right hand above her head to brush her hair. Which of the following nerves has most likely been iatrogenically injured? A. Accessory B. Ansa cervicalis C. Facial D. Hypoglossal E. Suprascapular

59 A. The spinal accessory nerve passes across the posterior triangle of the neck and innervates both the trapezius muscle and the sternocleidomastoid muscle for the respective side of the body. Upon surgical division of the nerve, the patient will lose the ability to raise the ipsilateral shoulder and will demonstrate weakness in turning the head to the opposite side. The trapezius will also lose tone and the shoulder will droop. The ansa cervicalis innervates strap muscles of the neck and, if cut, would not produce drooping of the shoulder. The facial nerve does not pass through any of the triangles of the neck; however, if it were divided, paralysis would result in the muscles of facial expression. The hypoglossal nerve innervates the intrinsic muscles of the tongue, plus the genioglossus, hyoglossus, and styloglossus and, if injured, would not result in any of the patient's symptoms. GAS 87; N 32; McM 29, 30, 32

A 2-day-old male infant has a noticeable gap in his upper lip. The diagnosis is a cleft lip. Failure of fusion of which of the following structures is the most likely cause of this anomaly? A. Lateral nasal and maxillary prominences/ processes B. Medial nasal prominences/processes C. Lateral nasal and medial nasal prominences/ processes D. Lateral prominences/processes E. Maxillary prominences/processes and the intermaxillary segment

6 E. The most common cause of cleft lip is failure of fusion of the maxillary process and the intermaxillary segment. Defects located between the lateral nasal prominences and the maxillary processes would affect the development of the nasolacrimal duct. Failure of fusion of the medial nasal prominences would produce a median cleft lip, a rare congenital anomaly. The lateral and median nasal processes both arise from the nasal placodes and do not undergo subsequent fusion. The lateral nasal prominences do not fuse with each other. GAS 1077; N 46; McM 80

A 20-year-old man is admitted to the emergency department with a stab wound in the superior region of his neck. A radiographic examination reveals that the wound has not affected any major structures. Physical examination reveals that the patient has lost sensation from the skin over the angle of the jaw. Which of the following nerves is most likely injured? A. Supraclavicular B. Transverse cervical C. Great auricular D. Greater occipital E. Lesser occipital

60 C. Four nerves participate in providing cutaneous supply to the neck. The nerves are the supraclavicular, great auricular, transverse cervical, and the lesser occipital. The area over the angle of the jaw is innervated by the great auricular nerve. It ascends from spinal segments from C2 and C3 and innervates the skin over the angle of the jaw and posteroinferior to the auricle of the ear. The transverse cervical also originates from the C2-3 spinal segments but passes anteriorly to innervate the anterior and lateral aspects of the neck. The lesser occipital nerve innervates skin in the area of the back of the neck and posterior occiput. The supraclavicular nerves originate from C3-4 and innervate the more inferior aspects of the neck, the upper deltoid region, and skin inferior to the clavicles. GAS 985-986; N 2; McM 29, 30, 31, 33

A 6-year-old boy, whose medical history includes a complicated delivery, has a continuously tilted head posture, with the right ear near the right shoulder and the face turned upward and to the left. Which of the following muscles was most likely damaged during birth? A. Anterior scalene B. Omohyoid C. Sternocleidomastoid D. Trapezius E. Platysma

61 C. Because of its size and vulnerable position during birth, the sternocleidomastoid muscle is injured more often than other muscles of the head and neck during birth especially if the delivery is difficult. When acting alone, the action of this muscle is to turn the head to the opposite side and bend it toward the ipsilateral shoulder. When using both muscles, the head will flex toward the chest. Therefore, the most likely muscle to have been injured here is the left sternocleidomastoid muscle. GAS 1024; N 29; McM 31, 32

A 35-year-old woman is admitted to the emergency department after a violent automobile crash. The patient's upper airway is obstructed with blood and mucus, and a midline tracheostomy inferior to the thyroid isthmus is performed. Which of the following vessels are most likely to be present at the site of incision and will need to be cauterized? A. Middle thyroid vein and inferior thyroid artery B. Inferior thyroid artery and inferior thyroid vein C. Inferior thyroid vein and thyroidea ima artery D. Cricothyroid artery and inferior thyroid vein E. Left brachiocephalic vein and inferior thyroid artery

62 C. The most likely structures one would encounter while performing a midline incision below the isthmus of the thyroid gland would be the inferior thyroid vein and the thyroidea ima artery. The inferior thyroid vein drains typically to the left brachiocephalic vein, which crosses superficially, just inferior to the isthmus. The thyroidea ima artery arises from the aortic arch, vertebral artery, or other source but is not a constant structure. The middle thyroid veins drain the thyroid gland to the internal jugular vein and are superior to the incision site. The inferior thyroid arteries branch from either subclavian artery and meet the thyroid gland at an oblique angle. They would not be ligated with a midline incision. The brachiocephalic veins are inferior to the site of incision. GAS 1019; N 31, 73, 76-78, 195, 208, 234; McM 30, 32, 37, 206

A 55-year-old woman is admitted to the hospital with difficulty swallowing. Physical examination reveals that the patient has episodes of severe headaches and frequently aspirates fluids when drinking them. A radiographic examination reveals a skull base tumor occupying the space behind the jugular foramen. Involvement of which of the following structures is most likely responsible for the findings in the patient? A. Ansa cervicalis B. Cervical sympathetic trunk C. External laryngeal nerve D. Hypoglossal nerve E. Vagus nerve

64 E. The vagus nerve exits the skull at the jugular foramen and is responsible for motor innervation to the smooth muscles of the trachea, bronchi, and digestive tract, in addition to the muscles of the palate, pharynx, larynx, and superior two thirds of the esophagus. The ansa cervicalis innervates the strap muscles of the neck, with the exception of the thyrohyoid muscle. The cervical sympathetic trunk does not enter into the jugular foramen; it runs behind the carotid sheath, parallel with the internal carotid artery; its carotid branch accompanies the artery into the carotid canal and carries sympathetic fibers to deep areas of the head. Damage to the external laryngeal nerve would result in paralysis of the cricothyroid muscle, presenting as an easily fatigued voice with hoarseness. Injury to the hypoglossal nerve would result in protrusion of the tongue toward the affected side and moderate dysarthria. GAS 894-895, 900, 902, 1034; N 12, 26, 32-34, 42, 55, 61, 71-72, 76-78, 105, 115, 117-118, 126-128, 131-132, 136, 164, 203, 206-208, 224,

A 55-year-old woman is admitted to the hospital with severe hypertension. Examination reveals hypertension (190/110 mm Hg) and hypercholesterolemia (250 mg/dL). During physical examination she complains of headaches and dizziness. An arteriogram reveals 90% occlusion of both common carotid arteries. A carotid endarterectomy is performed and large atherosclerotic plaques are removed. During a postoperative physical examination on the right side, it was noted that her tongue deviated toward the right when she was asked to protrude it. Which of the following nerves was most likely injured during the procedure? A. Right glossopharyngeal B. Right hypoglossal C. Left hypoglossal D. Left lingual E. Left vagus

65 B. The hypoglossal nerve provides motor innervation to the muscles of the tongue, with the exception of the palatoglossus. Injury to the hypoglossal nerve would result in deviation of the tongue toward the affected side when the tongue is protruded (in this case the right side), due mainly to the unilateral contraction of left genioglossus, and moderate dysarthria. Injury to the glossopharyngeal nerve would result in loss of taste in the posterior third of the tongue and a loss of soft palate sensation and gag reflex on the affected side. The inferior alveolar nerve supplies the tissues of the chin and lower teeth. The lingual nerve conveys parasympathetic preganglionic fibers to the submandibular ganglion and general sensation and taste fibers for the anterior two thirds of the tongue. Injury to the vagus nerve would cause sagging of the soft palate, deviation of the uvula to the unaffected side, hoarseness, and difficulty in swallowing and speaking (GAS Fig. 8-257). GAS 894-895, 900, 901b, 902; N 12, 33-34, 42, 47, 50, 55, 59, 71-73, 75, 105, 115, 118-119, 129-130; McM 44, 67, 79, 81

A 34-year-old woman is admitted to the hospital with a large mass in her lower anterior neck. Ultrasound examination reveals a benign tumor of her thyroid gland. Twenty-four hours following a partial thyroidectomy, it was noted that the patient frequently aspirates fluid into her lungs. Upon examination it was determined that the area of the piriform recess above the vocal fold of the larynx was anesthetized. Which of the following nerves was most likely iatrogenically injured? A. External branch of the superior pharyngeal B. Hypoglossal C. Internal branch of the superior laryngeal D. Lingual E. Recurrent laryngeal

66 C. During removal of the tumor, the internal branch of the superior laryngeal nerve was injured. Injury to this nerve results in loss of sensation above the vocal cords, at the entrance to the larynx, and loss of taste on the epiglottis. Loss of sensation in the laryngeal vestibule can precipitate aspiration of fluid into the larynx, trachea, and lungs. The pharyngeal nerve from the vagus nerve supplies motor innervation to the muscles of the pharynx, except the stylopharyngeus (glossopharyngeal nerve). Injury to the hypoglossal nerve would result in protrusion of the tongue toward the affected side and moderate dysarthria. The lingual nerve conveys parasympathetic preganglionic fibers to the submandibular ganglion and general sensation and taste fibers for the anterior two thirds of the tongue. The recurrent laryngeal provides sensory fibers to the larynx below the vocal cords and motor fibers to all of the muscles of the larynx except for the cricothyroid. GAS 1068; N 61, 65, 67, 69, 71, 76-78, 80, 82, 127; McM 47, 78

A 46-year-old woman is admitted to the hospital with a large mass in her lower anterior neck. Ultrasound examination suggests a benign tumor of her thyroid gland, which is confirmed with a biopsy. During the procedure to remove the tumor the superior thyroid artery is identified and used as a landmark in order not to damage its small companion nerve. Which of the following nerves is most likely to accompany the superior thyroid artery? A. Cervical sympathetic trunk B. External branch of the superior laryngeal C. Inferior root of the ansa cervicalis D. Internal branch of the superior laryngeal E. Recurrent laryngeal

68 B. The external branch of the superior laryngeal nerve courses together with the superior thyroid artery for much of its route. The cervical sympathetic trunk is located more laterally and quite posteriorly to this location. The inferior root of the ansa cervicalis is located more superficially in the anterior neck. The internal branch of the superior laryngeal nerve takes a route superior to that of the external branch and the superior thyroid artery and would be unlikely to be injured in this case. The recurrent laryngeal nerve terminates inferiorly, passing into the larynx in relation to the inferior thyroid artery or its branches. GAS 1068; N 71, 76-78, 82, 127; McM 47, 78

A 3-year-old girl ruptured her tympanic membrane (eardrum) when she inserted a pencil into her external ear canal. Physical examination revealed pain in her ear and a few drops of blood in the external auditory meatus. There was the concern that there might possibly have been an injury to the nerve that principally innervates the external surface of the tympanic membrane. Which of the following tests is most likely to be performed during physical examination to check for injury to this nerve? A. Check the taste in the anterior two thirds of the tongue B. Check the sensation to the pharynx and palate C. Check if there is paraesthesia at the TMJ D. Check for sensation in the larynx E. Check for sensation in the nasal cavity

69 C. The external surface of the tympanic membrane is innervated primarily by the auriculotemporal nerve, a branch of the mandibular division of the trigeminal nerve. Damage to this nerve would additionally result in painful movements of the TMJ because this joint receives innervation from the same nerve. Taste in the anterior two thirds of the tongue is supplied by the facial nerve and would be unaffected in this injury. (The chorda tympani could be injured, but its superior location on the medial side would make this unlikely.) The sensory innervation of the nasal cavity is supplied by the ophthalmic and maxillary divisions of the trigeminal nerve and would be unaffected by injury to the tympanic membrane. Sensory innervation to the larynx is provided by the vagus nerve, whereas the pharynx receives sensory fibers from the glossopharyngeal and vagus nerves. The palate is supplied by the maxillary divisions of the trigeminal nerve and would be unaffected by this injury. GAS 912, 915, 985-986, 1104; N 2, 18, 46, 49-51, 71-72, 123, 126, 135; McM 29, 39-40, 59, 75-76

A 3-day-old male infant has a noticeably small mandible. A computed tomography (CT) scan and physical examinations reveal hypoplasia of the mandible, cleft palate, and defects of the eye and ear. Abnormal development of which of the following pharyngeal arches will most likely produce such symptoms? A. First arch B. Second arch C. Third arch D. Fourth arch E. Sixth arch

7 A. The listed symptoms are typical of first pharyngeal (brachial) arch syndrome because the first arch normally gives rise to muscles of mastication, mylohyoid, anterior belly of the digastric, tensor tympani, tensor veli palatini, malleus, and the incus. Abnormal development of the second arch would affect the muscles of facial expression, the stapes, and parts of the hyoid bone. Abnormal development of the third pharyngeal arch would affect only the stylopharyngeus muscle and parts of the hyoid bone. Abnormal development of the fourth and sixth arch would affect various muscles and cartilages of the larynx and pharynx and would not produce the hypoplastic mandible characteristic of first arch syndrome. GAS 977, 961, 1009; N 48-49, 74, 94; McM 35, 39, 61

A 27-year-old woman is admitted to the emergency department after she was thrown from a motor scooter. Radiographic evaluation reveals a type I LeFort fracture and comminuted fracture of the mandible and TMJ. Despite reconstructive surgery, the patient develops hyperacusis (sensitivity to loud sounds) due to facial nerve paralysis. Which of the following muscles is most likely paralyzed? A. Posterior belly of digastric B. Stapedius C. Tensor tympani D. Stylohyoid E. Cricothyroid

70 B. Both the stapedius and tensor tympani normally function to dampen movements of the middle ear ossicles, thereby muting sound and preventing hyperacusis. A stapedius would be the source of hyperacusis in this problem because it receives its innervation from the facial nerve. The tensor tympani receives motor innervation from the mandibular division of the trigeminal nerve. The posterior belly of the digastric and the stylohyoid receive innervation from the facial nerve, but their paralysis would not cause hyperacusis. Damaged innervation of the cricothyroid, which is supplied by the external branch of the superior laryngeal nerve, would not result in hyperacusis. GAS 963, 969-971; N 55, 96, 119, 124, 151; McM 60-61

A 43-year-old man is admitted to the emergency department with a fracture of the base of his skull. A thorough physical examination reveals that a number of structures have been injured, suggesting that the right greater petrosal nerve has been injured. Which of the following symptoms needs to be identified during physical examination to confirm the diagnosis of greater petrosal nerve injury? A. Partial dryness of the mouth due to lack of salivary secretions from the submandibular and sublingual glands B. Partial dryness of the mouth due to lack of salivary secretions from the parotid gland C. Dryness of the right cornea due to lack of lacrimal gland secretion D. Loss of taste sensation from the right anterior two thirds of the tongue E. Loss of general sensation from the right anterior two thirds of the tongue

71 C. The greater petrosal nerve carries parasympathetic fibers that are involved in the innervation of the lacrimal gland, as well as the mucosal glands of the nose, palate, and pharynx. As a result, an injury to the right greater petrosal nerve would be expected to result in decreased lacrimal secretions for the right eye. The sublingual and submandibular glands receive their parasympathetic fibers from the facial nerve via the chorda tympani and the lingual nerve. They would be unaffected by this lesion. The parotid gland receives its parasympathetic secretory innervation from the glossopharyngeal nerve via the lesser petrosal and auriculotemporal nerves and would be unaffected. Taste to the anterior tongue is provided by the facial nerve via the chorda tympani, and general sensation to the anterior tongue is provided by the mandibular division of the trigeminal nerve via the lingual nerve. GAS 969, 988, 997, 1104, 1105; N 13, 39, 53, 55, 61, 88, 96, 124-126, 132, 134, 136; McM 44, 61, 79

A 12-year-old girl is admitted to the emergency department with a middle ear infection. Physical examination reveals a long history of chronic middle ear infections that have produced a lesion in the tympanic plexus in the middle ear cavity. Since the preganglionic parasympathetic fibers that pass through the plexus have been lost, which of the following conditions will be detectable during physical examination? A. Diminished mucus in the nasal cavity B. Diminished mucus on the soft palate C. Diminished saliva production by the parotid gland D. Diminished saliva production by the submandibular and sublingual glands E. Diminished tear production by the lacrimal gland

72 C. Parasympathetic innervation of the parotid gland is provided by axons carried by the glossopharyngeal nerve that emerge from the tympanic plexus of the middle ear as the lesser petrosal nerve. These preganglionic parasympathetic fibers terminate at synapses in the otic ganglion, which supplies the secretory parasympathetic innervation to the parotid gland. Glandular secretions of the nasal cavity, soft palate, and lacrimal gland all receive parasympathetic innervation from the fibers of the greater petrosal nerve and would remain intact following a tympanic plexus lesion. Axons for secretory innervation to the sublingual and submandibular glands are carried by the facial nerve, then course through the chorda tympani, before synapsing in the submandibular ganglion, with postganglionic fibers eventually reaching the glands via the lingual nerve (GAS Figs. 8-116 and 8-121). GAS 964, 989; N 12-13, 50, 88, 95-96, 123-124, 126, 135; McM 44, 78-79

A 38-year-old woman is admitted to the dental clinic with acute dental pain. The attending dentist found penetrating dental caries (tooth decay) affecting one of the mandibular molar teeth. Which of the following nerves would the dentist need to anesthetize to treat the caries in that tooth? A. Lingual B. Inferior alveolar C. Buccal D. Mental E. Mylohyoid

73 B. The inferior alveolar branch of the mandibular division of the trigeminal nerve provides sensory innervation to the mandibular teeth and would require anesthesia to abolish painful sensation. The lingual nerve provides taste and sensation to the anterior two thirds of the tongue and carries general sensory fibers, taste fibers, and parasympathetic fibers. It does not provide sensory innervation to the teeth. The buccal nerve provides sensory innervation to the inner surface of the cheek. The mental nerve is the distal continuation of the inferior alveolar nerve as it exits the mental foramen of the mandible and does not affect the teeth. The nerve to the mylohyoid is a motor branch of the inferior alveolar nerve that supplies the mylohyoid and the anterior belly of the digastric. GAS 894, 896, 897-898, 901-902, 989; N 18, 47, 49-51, 58, 61, 71, 123, 132, 135; McM 42, 59, 75-76, 81

A 59-year-old man is admitted to the emergency department with acute pain on his mandible. An MRI examination reveals an acute inflammation of the temporomandibular joint due to arthritis. Which of the following muscles will most likely be affected by the inflammatory process of this joint? A. Temporalis B. Medial pterygoid C. Masseter D. Lateral pterygoid E. Buccinator

74 D. Part of the lateral pterygoid muscle has its insertion on the articular disc within the TMJ and would be most affected by the inflammation of this joint. The temporalis muscle inserts upon the coronoid process and retracts the jaw. The medial pterygoid muscle extends from the medial surface of the lateral pterygoid plate to the mandible and functions in elevation of the jaw. The masseter extends from the zygomatic arch to the lateral ramus of the mandible and elevates the jaw. The buccinator pulls back the angle of the mouth and flattens the cheek. GAS 977, 983, 985; N 6, 42, 49-51, 151; McM 10, 19, 42-44

A 56-year-old woman complains of diplopia (double vision) when walking down stairs. A lesion of which of the following nerves is most likely responsible for this patient's complaint? A. Optic B. Oculomotor C. Abducens D. Trochlear E. Frontal

75 D. The trochlear nerve innervates the superior oblique muscle, which acts to move the pupil downward and laterally. It is the only muscle that can depress the pupil when the eye is adducted. When an individual walks down stairs, this eye motion is initiated, and diplopia results if it is not functioning properly. The optic nerve provides vision, and a lesion of this nerve would not result in diplopia when an affected individual walks down the stairs, but rather diminished vision or blindness. The oculomotor nerve supplies the superior, inferior, and medial rectus as well as the inferior oblique. Overall, innervation from the oculomotor nerve results in upward and inward movements of the eye, and a lesion of this nerve would not induce diplopia in an individual walking down stairs. The abducens nerve innervates the lateral rectus muscle, which abducts the eye, and damage would not induce the diplopia presented in this problem. The frontal nerve is a branch of the ophthalmic division of the trigeminal nerve and provides sensory innervation to the forehead. GAS 894-895, 897, 901b, 902, 943-944; N 55, 85-86, 88, 105, 115-119, 122, 145; McM 53, 59, 74, 82

A 43-year-old man is admitted to the hospital complaining of diplopia (double vision) when walking down stairs. During physical examination of the extraocular muscles the patient experiences diplopia, and when he is asked to turn his right eye inward toward his nose and look down, he is able to look inward but not down. Which nerve is most likely involved? A. Abducens B. Nasociliary C. Oculomotor, inferior division D. Oculomotor, superior division E. Trochlear

76 E. The superior oblique muscle turns the pupil downward from the adducted position. Inability to perform this motion, in conjunction with diplopia when walking down stairs, indicates damage to the trochlear nerve. The abducens innervates the lateral rectus, resulting in abduction of the eye. The oculomotor nerve supplies the superior, inferior, and medial rectus as well as the inferior oblique muscles. Overall, innervation from the oculomotor nerve results in upward and downward movements of the eye. Damage to this nerve would not induce diplopia when an affected individual walks down stairs. In addition, inability to gaze downward in the adducted position does not indicate oculomotor nerve damage. In this position the oculomotor nerve would be responsible for upward movement. The nasociliary nerve is a sensory nerve originating from the ophthalmic branch of the trigeminal nerve. GAS 894-895, 897, 901b, 902, 943-945; N 55, 85-86, 88, 105, 115-119, 122, 146; McM 53, 59, 74, 82

A 44-year-old woman is being treated for Raynaud's disease. A sympathetic blocking drug is administered in high doses. Which of the following conditions will most likely be expected to occur as an adverse effect of the drug? A. Exophthalmos and dilated pupil B. Enophthalmos and dry eye C. Dry eye and inability to accommodate for reading D. Wide open eyelids and loss of depth perception E. Ptosis and miosis

77 E. Ptosis and miosis occur in response to blocking of sympathetic innervation. Ptosis (drooping of the eyelid) results from lack of innervation of the superior tarsal muscle (of Müller), and miosis (pupillary constriction) results from unopposed parasympathetic innervation of the pupil. A dilated pupil would not occur because this requires the action of the sympathetically innervated dilator pupillae. Dry eye would occur due to lacrimal gland insufficiency, but because this is mediated by parasympathetic fibers, it would remain unaffected in this case. The same holds true for the parasympathetically mediated accommodation pathway. Depth perception involves the visual pathway and is not mediated by the sympathetic system. GAS 928-931; N 83, 132; McM 56, 81

A 47-year-old woman is admitted to the hospital with signs of cavernous sinus thrombosis. Radiographic examination reveals a pituitary tumor involving the cavernous sinus, confirming the initial diagnosis. During physical examination it is suspected that the right abducens nerve of the patient has been damaged by the tumor. In which direction will the physician most likely ask the patient to turn her right eye to confirm the abducens nerve damage, assuming she is unable to perform this task? A. Inward B. Outward C. Downward D. Down and out E. Down and in

78 B. The right abducens nerve innervates the right lateral rectus, which mediates outward movement (abduction) of the right eye. Inward movement is accomplished by the medial rectus, supplied by the oculomotor nerve. Downward movement in the midline is accomplished by joint activation of the superior oblique and inferior rectus muscle. Downward movement of the pupil from the adducted position is a function of the superior oblique alone, which is supplied by the trochlear nerve. Down and out motion is mediated by the combined actions of the lateral rectus and inferior rectus muscles, which are innervated by the abducens and oculomotor nerves. Downward movement of the pupil from a forward gaze is a result of combined actions of inferior rectus and superior oblique muscles, supplied by oculomotor and trochlear nerves, respectively. GAS 894-895, 898, 901b, 902, 944; N 13, 55, 85-86, 88, 105, 115, 118, 122; McM 67, 74, 77, 80

A 5-day-old male infant has an abnormally large head. A CT scan examination reveals enlarged lateral and third ventricles but a fourth ventricle of normal size. Stenosis of the cerebral aqueduct (of Sylvius) is suspected. Which of the following conditions will be characteristic of these symptoms? A. Nonobstructive hydrocephalus B. Anencephaly C. Obstructive hydrocephalus D. Meroanencephaly E. Holoprosencephaly

8 C. Obstructive hydrocephalus, in this case resulting from obstruction of the cerebral aqueduct (of Sylvius), refers to a condition in which flow of cerebrospinal fluid (CSF) is obstructed within the ventricular system. This leads to pressure increasing in the CSF upstream from the obstruction, expanding the lateral and third ventricles. Nonobstructive hydrocephalus is due to either excessive CSF production or ineffective CSF reabsorption. This would lead to enlargement of all ventricular chambers. Anencephaly, also known as meroanencephaly, is a partial absence of the brain and is due to defective closure of the anterior neuropore. Holoprosencephaly is a failure of cleavage of the forebrain and would result in a single fused ventricle. GAS 877-878; N 147; McM 72, 81

A 32-year-old woman is admitted to the hospital with headaches and dizziness. During physical examination it is noted that the patient has partial ptosis (drooping eyelid). Which of the following muscles is most likely paralyzed? A. Orbicularis oculi, lacrimal part B. Orbicularis oculi, palpebral part C. Levator palpebrae superioris D. Superior oblique E. Superior tarsal (of Müller)

80 E. The superior tarsal muscle (of Müller), innervated by sympathetics, is smooth muscle that assists in elevating the eyelids and maintaining this position. Loss of sympathetic innervation will result in partial ptosis of the eyelid. The superior oblique, innervated by the trochlear nerve, moves the pupil downward from the adducted position (for example, as when the right eye gazes down toward the left foot). To test the trochlear nerve, ask the patient to look with each eye toward the tip of the nose. The orbicularis oculi, innervated by the facial nerve, is responsible for closure of the eye. The palpebral part closes the eyelids ordinarily; the lacrimal part contracts when the eye is closed more forcibly, resulting in increased tear movement across the globe (perhaps to flow down the cheeks). Damage to the levator palpebrae superioris, innervated by the oculomotor nerve, would result in complete, rather than partial, ptosis. GAS 928-931; N 83; McM 81

A 16-year-old boy is admitted to the hospital with fever, a confused mental state, and drowsiness. During physical examination it is noted that the boy suffers from severe acne. Radiologic examination reveals cavernous sinus thrombosis. Which of the following routes of entry to the cavernous sinus would most likely be responsible for the infection and thrombosis? A. Carotid artery B. Mastoid emissary vein C. Middle meningeal artery D. Ophthalmic vein E. Parietal emissary vein

81 D. Cavernous sinus thrombosis can often result from squeezing pimples or other infectious processes located around the danger area of the face, which includes the area of the face directly surrounding the nose. This physical pressure has the potential to move infectious agents from the pimple into the ophthalmic vein, which then carries it to the cavernous sinus. The pterygoid venous plexus and ophthalmic vein both communicate with the cavernous sinus and therefore offer a route of travel for the spread of infection, but the path provided by the superior ophthalmic vein is a more direct route. Additionally, the superior ophthalmic vein receives blood supply from the supraorbital, supratrochlear, and angular veins that supply the area around the nose and lower forehead. (Venous blood in the head can flow in either direction because these veins do not possess valves.) The emissary veins communicate between the venous sinuses and the veins of the scalp and would therefore not be involved in the spread of infection between the nose and cavernous sinus. The middle meningeal artery courses between the dura and periosteum, whereas the carotid artery, specifically the ICA, traverses through the cavernous sinus and provides origin to the ophthalmic artery. As with the middle meningeal artery, the carotid artery would not offer a route of communication between the area of infection and the cavernous sinus (GAS Fig. 8-65). GAS 887-888, 920, 934; N 13, 73, 85, 87, 93, 105; McM 47, 51

A 68-year-old man is admitted to the emergency department after an acute cerebral vascular accident (stroke). Radiologic studies reveal that the primary damage was to the anterior inferior cerebellar artery, resulting in a small hemorrhage of the artery at its origin from the main trunk. Which of the following nerves will most likely be immediately affected by the hemorrhage? A. Optic nerve B. Oculomotor nerve C. Trochlear nerve D. Trigeminal nerve E. Abducens nerve

82 E. The anterior inferior cerebellar artery (AICA) is a major supplier of the anterior inferior portion of the cerebellum. Nerves located in close proximity would likely be affected by hemorrhage of this artery. The optic, oculomotor, and trochlear nerves are all associated with the midbrain region and would likely not suffer any damage with a possible hemorrhage. The trigeminal nerve is situated in the pons and is thus located too far rostrally to be affected. The abducens nerve is situated at the pontomedullary junction and is therefore most likely to be damaged following hemorrhage of the AICA. GAS 882; N 137, 139-142, 144, 167; McM 68

A 5-year-old boy is admitted to the hospital with otitis media. Otoscopic examination reveals a bulging and inflamed eardrum. It is decided to incise the tympanic membrane to relieve the painful pressure and allow drainage of the infectious fluid associated with otitis media. Which of the following is the best location to make an opening (myringotomy) for drainage? A. The anterior superior quadrant of the eardrum B. The posterior superior quadrant of the eardrum C. Directly through the site of the umbo D. The anterior inferior quadrant of the eardrum E. A vertical incision should be made in the eardrum, from the 12 o'clock position of the rim of the eardrum to the 6 o'clock position of the rim

83 D. The anterior inferior quadrant of the eardrum is the only portion of the tympanic membrane that would allow for an incision with minimal or no damage to adjacent important structures. Incision in the anterior and posterior superior quadrants of the eardrum would likely damage the malleus, which is situated immediately superior and medially to the tympanic membrane. The umbo is situated in close proximity to the handle of the malleus and might be damaged during incision. A vertical incision through the eardrum would almost certainly damage the malleus of the middle ear. Damage to the malleus from surgical incision would interfere with the auditory conduction through the middle ear cavity, and this should be avoided to prevent conductive hearing loss. GAS 956-958, 971; N 94-96, 98; McM 44, 60-61

A 56-year-old man is diagnosed with an extradural tumor in the posterior cranial fossa. When the patient protruded his tongue during physical examination, the tongue deviated to the right. Which of the following muscles and nerves are most likely injured? A. Right hypoglossal nerve and right genioglossus B. Left hypoglossal nerve and left genioglossus C. Right hyoglossus and left styloglossus D. Right geniohyoid and first cervical nerve E. Contralateral vagus and hypoglossal nerves

84 A. The hypoglossal nerve innervates the muscles of the tongue and is therefore directly involved in alteration of shape and movement of the tongue. A lesion in this nerve would cause deviation of the tongue toward the injured side, which could be observed upon protrusion of the tongue. The genioglossus is the major muscle involved in protrusion of the tongue. The genioglossus muscles arise from the inside of the mandible and pass posteriorly to insert into the deep aspect of the tongue. When the genioglossus muscles contract, they pull the tongue forward, and out of the mouth, in protrusion. If one genioglossus is paralyzed, it acts like a brake on one side of the tongue when the tongue is pulled forward, causing the tip of the tongue to point to the nonmoving side. The styloglossus muscle is responsible for retraction and elevation of the tongue. GAS 894-895, 900, 901, 902; N 12, 33-34, 42, 47, 50, 55, 59, 71-73, 75, 105, 115, 118-119, 129-130; McM 44, 67, 79, 81

A 62-year-old man is admitted to the hospital with blurred vision. Taking of his history and performance of a physical examination reveals a long history of gradual loss of his visual field. The intraocular pressure is high, and a diagnosis of glaucoma is made. Which of the following spaces first receives the aqueous humor secreted by the epithelium of the ciliary body? A. Anterior chamber B. Posterior chamber C. Pupil D. Vitreous E. Lacrimal sac

85 B. The posterior chamber receives ciliary body secretions first. The ciliary body produces aqueous humor and is located in the posterior chamber. Increased production of fluid from this site would cause an increase in intraocular pressure if drainage is inadequate. The iridoscleral angle of the anterior chamber is the location of drainage of the aqueous humor; therefore, a blockage of drainage in this location can cause increased intraocular pressure. The pupil is the connection between the anterior and posterior chamber; a collection of fluid does not occur here, for this is simply an aperture to allow light onto the retina. The vitreous body is not directly connected to the production of aqueous humor. The lacrimal sac is the upper dilated end of the nasolacrimal duct and opens up into the inferior meatus of the nasal cavity. The nasolacrimal duct has nothing to do with increased intraocular pressure. GAS 947-948, 950; N 89-90; McM 54

A 17-year-old girl is admitted to the hospital with tonsillitis. A tonsillectomy is performed and the patient complains postoperatively of ear pain. Which of the following nerves was most likely injured during the surgical procedure? A. Auriculotemporal B. Lesser petrosal C. Vagus D. Glossopharyngeal E. Chorda tympani

86 D. The glossopharyngeal nerve mediates general somatic sensation from the pharynx, the auditory tube, and from the middle ear. Painful sensations from the pharynx, including the auditory tube, can be referred to the ear by this nerve, as in this case of tonsillectomy. The auriculotemporal nerve supplies skin of the auricle and tympanic membrane and scalp. This nerve would not be involved directly or indirectly in the operation. The lesser petrosal nerve contains preganglionic parasympathetic fibers that run in the glossopharyngeal and tympanic nerves before synapsing in the otic ganglion. The vagus nerve mediates general somatic afferent supply to the auricle and external acoustic meatus; stimulation of the meatus can trigger a gag reflex or coughing reflex. The chorda tympani mediates taste for the anterior two thirds of the tongue. GAS 894-895, 899-900, 901, 902; N 34, 42, 55, 61, 71-72, 105, 115, 117-118, 124, 126-127, 131-132, 134-136, 207; McM 44, 67, 78

A 49-year-old woman is admitted to the hospital with headaches and dizziness. Radiologic examination reveals a tumor in the jugular canal. Upon physical examination, when the right side of the pharyngeal wall is touched with a tongue depressor, the uvula deviates to the left and the left pharyngeal wall contracts upward. When the left pharyngeal wall is touched, the response is similar. Which of the following nerves is most likely to have been injured by the tumor? A. Right glossopharyngeal B. Left glossopharyngeal C. Right mandibular D. Left hypoglossal E. Right vagus

87 E. A tumor of the jugular canal would likely affect the glossopharyngeal, vagus, and accessory nerves as they exit the cranium through the jugular foramen. The uvula deviates toward the unaffected side of the pharyngeal muscles because of the pull of the unopposed levator veli palatini. In this case, the uvula deviates to the left, indicating that the left palatal muscles are unaffected whereas the right muscles are not working properly. The pharyngeal wall on the left side is also drawn upward by the nonparalyzed stylopharyngeus, supplied by the left glossopharyngeal nerve. The pharyngeal constrictor muscles, as well as muscles of the palate, are all innervated by the vagus nerve, except for the tensor veli palatini, which is supplied by the mandibular division of the trigeminal nerve. The right mandibular nerve (of the mandibular division of trigeminal nerve) provides sensory innervation to the face and motor supply to the masticatory muscles and does not innervate the muscles of the pharynx. The left hypoglossal nerve innervates the intrinsic and extrinsic muscles of the left side of the tongue. Compression or injury of this nerve would not lead to uvula deviation. GAS 894-895, 900, 902, 1034; N 12, 26, 32-34, 42, 55, 61, 71-72, 76-78, 105, 115, 117-118, 126-128, 131-132, 136, 164, 203, 206-208, 224, 229, 236, 303-304, 318; McM 47, 53, 67-69, 78, 207

A 45-year-old man is admitted to the emergency department with severe dyspnea. During physical examination there is swelling in the floor of his mouth and pharynx and his airway is nearly totally occluded. In addition, there is a swelling in his lower jaw and upper neck. His physical history indicates that one of his lower molars was extracted a week ago and he had been feeling worse every day since. Which of the following conditions will be the most likely diagnosis? A. Quinsy B. Torus palatinus C. Ankyloglossia D. Ranula E. Ludwig's angina

88 E. An infection of the submandibular space is usually the result of a dental infection in the mandibular molar area in the floor of the mouth (Ludwig's angina). If the patient is not treated with antibiotics promptly, the pharyngeal and submandibular swelling can lead to asphyxiation. Quinsy, also known as peritonsillar abscess, is a pus-filled inflammation of the tonsils that can occur due to tonsillitis. Ankyloglossia, which is also known as tongue-tie, is a congenital defect that results in a shortened lingual frenulum that restricts movement of the tongue. The affected person will usually have a speech impediment. Torus palatinus is a benign bony growth on the hard palate; a torus mandibularis is a similar growth on the inside of the mandible. Such growths are usually benign and would not typically cause pain. A ranula is a mucocele found on the floor of the mouth, often resulting from dehydration in older individuals, with coagulation (inspissation) of salivary secretions. It can be caused by acute local trauma; however, they are usually asymptomatic. GAS 988, 1103; N 27-29; McM 18, 30, 32-33, 40

A 5-year-old girl is admitted to the hospital with an upper respiratory tract infection. During physical examination her sense of hearing appears to be poor. Her right ear is painful, and upon otoscopic examination a golden brown fluid can be observed through the tympanic membrane. Which is the most likely direct route for the spread of an infection from the upper respiratory tract to the middle ear cavity? A. Pharyngotympanic tube B. Choanae C. Nostrils D. Facial canal E. Internal acoustic meatus

89 A. The auditory (eustachian or pharyngotympanic) tube is a mucosal-lined tube that provides a direct connection from the nasopharynx to the middle ear cavity. A respiratory infection can travel from the upper respiratory tract to the oropharynx or nasopharynx and then on into the middle ear via the auditory tube. The choanae are paired openings from the nasal cavity into the nasopharynx and do not connect with the auditory tube or the middle ear. The facial canal and the internal acoustic meatus are passages for facial and vestibulocochlear nerves, respectively. Neither of these is a likely site for the spread of infection from the upper respiratory tract to the middle ear. GAS 954, 961, 966, 1046-1048, 1090; N 36, 42, 49, 57, 64-69, 94-96, 98, 100, 126; McM 52, 58, 60-61

A 3-month-old male infant has a lump in his neck. A biopsy of the lump shows it to be thymic tissue. Based on embryonic origin, which of the following additional structures is most likely to have an ectopic location? A. Jugulodigastric lymph node B. Lingual tonsil C. Parathyroid gland D. Submandibular gland E. Thyroid gland

9 C. Both the inferior parathyroid glands and the thymus are derived from the third pharyngeal (brachial) pouch. Therefore, an ectopic thymus is likely to be associated with ectopic parathyroid tissue, indicating abnormal development of the third pharyngeal pouch. The lingual tonsil develops from an aggregation of lymph nodules on the tongue and is not associated with development of the thymus. The submandibular gland develops from endodermal buds in the floor of the stomodeum and is not associated with development of the thymus. The thyroid gland arises from an outpocketing of the floor of the primitive oral cavity, descending along the route of the thyroglossal duct, and it is not associated with development of the thymus. Development of the lymph nodes is also not associated with development of the thymus. GAS 1022; N 78; McM 30

A 54-year-old man is admitted to the hospital with severe pain in his nasal cavity. Radiographic examination reveals a carcinoma in his nasal cavity. In which of the following locations would the carcinoma block the hiatus of the maxillary sinus? A. Inferior meatus B. Middle meatus C. Superior meatus D. Nasopharynx E. Sphenoethmoidal recess

90 B. The maxillary sinus drains via the middle meatus, specifically into the semilunar hiatus. The middle meatus and semilunar hiatus are located under the middle nasal concha. The inferior meatus drains the lacrimal secretions carried by the nasolacrimal duct, whereas the superior meatus drains the posterior ethmoidal and sphenoid sinuses. The nasopharynx and sphenoethmoidal recess are not situated in close proximity to the maxillary sinus and are therefore not involved in its drainage (GAS Fig. 8-235). GAS 1069-1071; N 36-37, 43-44; McM 3, 12, 16, 44, 58

A 54-year-old man is diagnosed with an aneurysm of the basilar artery close to the cavernous sinus. An anterior approach to the sella turcica through the nasal cavity is performed. Through which of the following routes is the surgeon most likely to gain access to the cranial cavity? A. Cribriform plate B. Cavernous sinus C. Frontal sinus D. Maxillary sinus E. Sphenoidal sinus

91 E. The sphenoidal sinus provides the most direct access to the pituitary gland, which is situated directly above this sinus. Neither the frontal sinus nor maxillary sinus has any direct communication with the interior of the cranial vault and would therefore not allow the surgeon a potential access point to the pituitary gland. The cribriform plate could offer a point of entry into the cranium; entry at that site would lead to damage of the olfactory cells and nerve, but it would also lead to entry into the subarachnoid space, with leakage of CSF and potential meningitis. The cribriform plates are also located too far anteriorly from the pituitary gland. The cavernous sinus is situated within the cranial vault and surrounds the pituitary gland; it is not a site for surgical entrance to the cranial cavity. GAS 1075, 1076; N 38, 39; McM 12, 17, 25, 43, 47, 58, 73

A 10-year-old girl is admitted to the hospital with tonsillitis. A tonsillectomy is performed and the tonsils are removed. On physical examination 1 week later the patient has absence of the gag reflex on the left when the posterior part of the tongue is depressed. The sensory portion of which of the following nerves was most likely injured? A. Facial B. Glossopharyngeal C. Mandibular D. Maxillary E. Hypoglossal

92 B. The gag reflex is composed of both an afferent and an efferent limb. These reflexes are mediated by the glossopharyngeal and vagus nerves, respectively. Together, the glossopharyngeal and vagus nerves are responsible for the contraction of the muscles of the pharynx involved in the gag reflex. In this case the glossopharyngeal nerve was injured when the tonsils were excised, resulting in the loss of the sensory side of the reflex. The mandibular and maxillary nerves are part of the trigeminal nerve and are thus largely associated with the sensory supply of the face, sinuses, and oral cavity. The hypoglossal nerve innervates most of the muscles of the tongue. The facial nerve is involved with taste of the anterior two thirds of the tongue; however, it does not mediate the gag reflex. GAS 1100; N 61; McM 29, 35, 36, 44, 67, 78

A 56-year-old woman has just undergone a complete thyroidectomy. After she recovers from the anesthesia a hoarseness of her voice is noted that persists for 3 weeks. Subsequent examination shows a permanently adducted vocal fold on the right side. Surgical trauma to the innervation of which of the following muscles is most likely to be responsible for the position of the right vocal fold? A. Aryepiglottic B. Posterior cricoarytenoid C. Thyroarytenoid D. Transverse arytenoids E. Vocalis

93 B. The recurrent laryngeal nerve is often at risk of being damaged during a thyroidectomy. Patients who have a transected or damaged recurrent laryngeal will often present with a characteristic hoarseness following surgery. The posterior cricoarytenoid is supplied by the recurrent laryngeal nerve and would thus be impaired following damage to the nerve. The posterior cricoarytenoid is the only muscle responsible for abduction of the vocal cords, and paralysis of this muscle would result in a permanently adducted position of the involved vocal cord. The other muscles listed are all adductors of the vocal cords, and paralysis of these would not lead to closure of the airway. GAS 1062; N 80, 81, 82; McM 47-49

A 45-year-old man with a complaint of ear pain, difficulty hearing in one ear, nose bleeds, and difficulty breathing through the nose is diagnosed with postnasal carcinoma. Otoscopic examination reveals fluid in the middle ear cavity. Hypertrophy of which of the following structures would be most likely to compromise the drainage of the auditory tube? A. Lingual tonsil B. Palatine tonsil C. Pharyngeal tonsil D. Superior constrictor muscle E. Uvula

94 C. The pharyngeal tonsil is situated in a slitlike space, the pharyngeal recess, in the nasopharynx behind the opening of the auditory (eustachian) tube, and a pharyngeal tonsil in this location can lead to blockage of the drainage of the auditory tube. The lingual tonsil is located in the posterior aspect of the tongue, whereas the palatine tonsil is contained within the tonsillar fossa between the palatoglossal and palatopharyngeal arches. An enlargement of the lingual tonsil or the palatine tonsil will not occlude the auditory tube due to their location in the oropharynx. The superior pharyngeal constrictor would not be involved in occlusion of the auditory tube because it is located more posteriorly. The uvula is drawn upward during deglutition and prevents food from entering the nasopharynx; it does not block the auditory tube. GAS 1047; N 36; McM 52, 360

While at summer camp, a 10-year-old boy develops severe pharyngitis and swollen tonsils. Infection may spread from the nasopharynx to the middle ear cavity along the derivative of which embryonic pharyngeal pouch? A. First B. Second C. Third D. Fourth E. Sixth

95 A. Infection can spread from the nasopharynx to the middle ear by way of the auditory tube, which opens to both spaces. The first pharyngeal pouch is responsible for formation both of the auditory tube and middle ear (tympanic) cavity. The second pharyngeal pouch persists as the tonsillar sinus and tonsillar crypts. The third pharyngeal pouch develops into the inferior parathyroid gland and thymus, whereas the fourth pharyngeal pouch forms the remainder of the parathyroid glands and the ultimobranchial body. The sixth pharyngeal pouch is not well defined and would therefore not contribute to the development of the auditory tube. GAS 961; N 94; McM 45-47, 49

A 25-year-old medical student discovers that his alarm has not worked and he is running late. Desperate to get to his biochemistry lecture in time, yet unbearably hungry, he quickly throws some bread in the toaster as he gets ready. Despite the toast burning a little, he eats it quickly and rushes out the door. The burnt parts of the toast scratch the roof of his mouth, leaving him with a stinging sensation there. What nerve is collecting this sensation from the hard palate? A. Posterior superior alveolar nerve B. Inferior alveolar nerve C. Lingual nerve D. Greater palatine nerve E. Lesser palatine nerve

96 D. The greater palatine nerve is responsible for the sensory innervation of the hard palate, or the hard part of the roof of the mouth. The lesser palatine nerve supplies the soft palate and palatine tonsil but is not involved in supply to the hard palate. The posterior superior alveolar nerve supplies multiple structures, including posterior portions of the gums, cheeks, and the upper posterior teeth. However, it is not involved in nerve supply to the hard palate. The inferior alveolar nerve has several branches, including the mental nerve, incisive branch, nerve to the mylohyoid, and inferior dental branch. These nerves do not supply the roof of the mouth and thus are not involved. The lingual nerve supplies taste and general sensation to the anterior two thirds of the tongue. GAS 995, 1112-1113; N 39, 41, 53-55, 57, 61, 71, 123, 132; McM 44, 58

A 32-year-old woman underwent a thyroidectomy. Two months postoperatively, it was observed that the patient had lost the ability to notice the presence of foreign objects in the laryngeal vestibule. Which of the following nerves was most likely injured? A. Internal laryngeal nerve B. External laryngeal nerve C. Glossopharyngeal nerve D. Hypoglossal nerve E. Recurrent laryngeal nerve

97 A. Damage to the internal laryngeal nerve would result in a general loss of sensation to the larynx above the vocal cords, leaving the patient with an inability to detect food or foreign objects in the laryngeal vestibule. The external laryngeal nerve and recurrent laryngeal nerve are both at risk during thyroidectomy. Damage to the recurrent laryngeal nerve would result in paralysis of all the laryngeal muscles except the cricothyroid; it would render the patient hoarse, with a loss of sensation below the vocal cords. Loss of the external laryngeal nerve would lead to paralysis of the cricothyroid muscle and vocal weakness. Injury to the hypoglossal nerve would result in weakness or paralysis of muscle movement of the tongue. GAS 1068; N 61, 65, 67, 69, 71, 76-78, 80, 82, 127; McM 49-50, 78

A 4-year-old boy suffering from ankyloglossia is brought to the speech therapist. The examining physician recommends that the child be admitted for operation by a pediatric surgeon. Which of the following surgical procedures would be most appropriate for this condition? A. Removal of pterygomandibular raphe B. Resection of the pterygoid hamulus bilaterally C. Cutting the lingual frenulum D. Repair of the palate E. Removal of the central segment of the hyoid bone

98 C. Ankyloglossia (tongue-tie) is characterized by a lingual frenulum that extends all the way to the tip of the tongue. This condition can cause problems with speech, feeding, and oral hygiene as a result of the low range of motion of the tongue. Ankyloglossia can be treated surgically by cutting the lingual frenulum. None of the other procedures described would treat this condition. GAS 1096; N 46, 56; McM 43, 47, 49, 59

An 8-year-old boy was suffering from a severe infection of the right middle ear. Within the course of a week, the infection had spread to the mastoid antrum and the mastoid air cells. The organisms did not respond to antibiotics, so the surgeon decided to perform a radical mastoid operation. Following the operation, it was noticed that the boy's face was distorted. The mouth was drawn upward to the left, and he was unable to close his right eye. Saliva tended to accumulate in his right cheek and dribble from the corner of his mouth. What structure was most likely damaged during the operation? A. Mandibular nerve B. Parotid duct C. Vagus nerve D. Facial nerve E. Glossopharyngeal nerve

99 D. Of the answer choices listed, the left facial nerve of the patient is the most likely to be damaged during the mastoidectomy. The facial nerve exits the skull via the stylomastoid foramen, just anterior to the mastoid process. A lesion of the facial nerve is likely to cause the symptoms described as a result of paralysis of the facial muscles. Depending upon the site of injury, the patient could also lose the chorda tympani branch of the facial nerve, leading to loss of taste from the anterior two thirds of the tongue ipsilaterally as well as loss of functions of the submandibular and sublingual salivary glands. The other nerves listed are not likely to be damaged during a mastoidectomy. GAS 970; N 96; McM 61, 39, 40, 77

A 5-day-old female infant was born with a laryngeal defect. The greater cornuae and the inferior part of the hyoid bone were absent at birth. Failure of development of which of the following embryonic structures most likely led to these defects? A. Maxillary prominence B. Mandibular prominence C. Second pharyngeal arch D. Third pharyngeal arch E. Fourth pharyngeal arch

D. The third pharyngeal arch gives rise to the greater cornuae and lower part of the hyoid bone, in addition to the stylopharyngeus muscle. The maxillary prominence is important in the development of the cheeks and upper lip. The mandibular prominence is important in development of the mandible. The second pharyngeal arch gives rise to the lesser cornu and upper part of the hyoid bone. The fourth pharyngeal arches, while extensively involved in development of the cartilage and muscles of the larynx, play no part in the development of the hyoid bone. GAS 1091; N 15; McM 48


संबंधित स्टडी सेट्स

Leccion 9 - Estructura: 9.1 "Llenar"

View Set

Biology 111-548- Chapter 5: Structure and Function of Plasma Membranes

View Set

Chapter 3: Construction Projects and Players

View Set

INFINITI VARIABLE COMPRESSION TURBO ENGINE

View Set

Ch. 40 Fluid, Electrolyte, and Acid-Base Balance

View Set

Chapter 17 - Fluid & Electrolytes

View Set